You are on page 1of 59

The Pericardium

Aletta Ann Frazier, MD


Pericardium: Anatomic Basics Pericardial Cyst [Figure 1]
• Fibrous anchors heart to mediastinum and • Congenital
chest wall • Right > left cardiophrenic angle
• Serous comprises visceral (“epicardium”) • Thin mesothelial capsule
and parietal layers • No enhancement
• Extends to proximal ascending aorta, • Homogeneous fluid density
superior vena cava, pulmonary trunk
• Thickness 0.7–2 mm on thin section CT
• Marginated by epicardial and mediastinal
fat

Superior Aortic Recess


• Pericardial sleeve encases ascending aorta
(AA) and proximal arch
• Crosses anterior aspect of ascending aorta
(AA) and main pulmonary artery (PA)
• Mimics aortic dissection,
lymphadenopathy, bronchogenic cyst Figure 1 A & B

Transverse Sinus Frontal chest X-ray shows rounded opacity contiguous


with right heart border. Companion axial CT image
• Posterior to ascending aorta (AA) and confirms a simple right cardiophrenic angle cyst
main pulmonary artery (PA) comprised of fluid density.
• Just above left atrium (LA)
• Communicates with superior aortic recess
• Mimics type A aortic dissection
Pericardial Cyst: MR Imaging [Figure 2]
Left Pulmonic Recess • T1-weighted SI = intermediate signal
• Pericardial recess contiguous with venous • Post-gadolinium = no enhancement
adventitia • T2-weighted SI = uniform high signal
• Between main pulmonary artery and left
superior pulmonary vein (LSV)
• Communicates with transverse pericardial
sinus

Oblique Sinus
• Posterior to left atrium (LA), partly
anterior to esophagus
• Separated from transverse sinus by
pericardial reflections
• Mimics bronchogenic cyst or esophageal
lesion
Figure 2 A & B
Pulmonary Venous Recess
Left image: Coronal T1-weighted MR image shows
• Pericardial recess contiguous with venous a mass of intermediate signal intensity adjacent to
adventitia the right atrium. Right image: Coronal T2-weighted
• Orifice of right inferior pulmonary vein MR image confirms the mass has homogeneous high
(RPV) and left atrium (LA) signal intensity suggestive of fluid content.
• Mimics bronchopulmonary lymph node or
lung mass

Congenital Absence of the Pericardium

Surgical Absence of the Pericardium


• Status post lung transplant
• Sharp indentation in PA window
• Abnormal left heart contour
• Lucency above left hemidiaphragm

Cardiac Imaging The Pericardium


1475
Diagnosis: Hemopericardium [Figure 3]
• A 53-year-old woman status post mitral
annuloplasty, on hemodialysis with
increasing dyspnea and enlarging cardiac
silhouette

Figure 4 A & B
Two selected images from cine MRI during diastole
show a large pericardial effusion and collapse of right
atrial and right ventricular walls (arrows) compatible
with tamponade.
Figure 3 A & B
Frontal chest X-ray shows newly enlarged cardiac
silhouette in patient status post mitral annuloplasty.
Companion axial CT demonstrates a large pericardial Type A aortic dissection with
collection of mixed fluid and soft tissue density. hemopericardium and tamponade
[Figure 5]
• Rapid filling of pericardial sac (no time for
Hemopericardium compensation)
• Pericardial effusion >35–45 HU • Inward bowing of anterior free wall of RA
• Rapid accumulation may lead to and RV
tamponade • Straightening or reversal of
• Scenarios interventricular septum
 Status post cardiac surgery • High contrast density in right atrium, IVC,
 Type A aortic dissection hepatic and renal veins
 Myocardial infarction
 Ruptured coronary bypass graft
 Coagulopathy
 Trauma (stab wound, line puncture)
 Neoplastic involvement

Pericardial Tamponade [Figure 4]


• Collapse of RV, RA free wall during
diastole
 Tachycardia compensates for low
stroke volume
• Beck’s triad
 Hypotension, jugular venous Figure 5 A & B
distension, muffled heart sounds Two contrast-enhanced axial CT images from a patient
• A clinical diagnosis, supported by imaging with Type A aortic dissection show high density
pericardial effusion (hemopericardium), inward bowing
findings of the right ventricular wall (arrow) and straightening
• Often due to rapid accumulation of fluid in of the interventricular septum (arrows) suggesting
pericardial sac tamponade.

The Pericardium Cardiac Imaging


1476
Pericarditis [Figure 6] Pericarditis – Potential Sequelae
• Clinical diagnosis • Fibrosis
 Sharp chest pain, altered with change • Adhesion
in position • Calcification
 Pericardial friction rub on auscultation • Constriction
 ST segment elevation on ECG
• Scenarios Pericardial Constriction [Figure 8]
 Idiopathic (most common diagnosis) • Reduced diastolic filling (symptomatic)
 Infection [viral, bacterial, tuberculosis  Constriction
(TB)] ™™ Conical ventricles, paradoxical
 Post-viral (2–3 weeks later) septal “bounce”, adherence, dilated
 Autoimmune disease atria
 Radiation  Venous congestion
 Myocarditis ™™ Large IVC, hepatic congestion,
 Renal failure (uremic) ascites
 Trauma • Pericardial thickening >4 mm (may be
• Inflammation of pericardial linings normal)
 Fibrin deposition on thickened visceral  Delayed enhancement on MRI
and parietal pericardium • Pericardial calcification (up to 50% cases)
 Shaggy, yellow fibrinous exudate on • Scenarios
gross inspection (“bread and butter”)  Healed or chronic pericarditis
• CT features  Prior cardiac surgery or infarct
 Smooth pericardial thickening  Thoracic radiotherapy
 Enhancement of linings
 Pericardial effusion
Figure 8

Selected
Figure 6
image,
Contrast-enhanced 4-chamber
axial CT image cine MRI from
from a patient a patient
with bacterial with chronic
pericarditis shows a tuberculous
pericardial effusion pericarditis
accompanied by and
smooth pericardial pericardial
thickening constriction
and lining demonstrates
enhancement. biventricular
conical
deformity
(arrows) and
A 66-year-old Man with tuberculous mildly dilated atria. Note pericardial effusion, fibrinous
stranding, and thickened pericardial linings.
pericarditis and abscess formation in
the adjacent epigastrium [Figure 7]

A 65-year-Old Man asymptomatic, with


No constrictive physiology [Figure 9]
• The presence of pericardial
calcification does not ensure
the diagnosis of constriction,
and the absence of calcification does not
exclude it

Figure 7 A & B

Contrast-enhanced axial CT image from a patient with


tuberculous pericarditis shows thickened, enhanced
pericardial linings with adhesions producing a complex,
multilocated pericardial effusion. Histology image
of tuberculous pericarditis demonstrates extensive
granuloma formation (yellow arrows).

Cardiac Imaging The Pericardium


1477
Diagnosis: Pericardial Lymphangioma
[Figure 11]

Figure 11 A, B & C
Figure 9 A & B Contrast-enhanced
Two contrast-enhanced axial CT images from a axial CTA, a
patient with prominent pericardial calcifications but no selected image
constrictive physiology. Pericardial calcification does from companion
not always correspond to constriction. cine SSFP MRI,
and an oblique 3D
CTA reconstruction
image show a
partly calcified
Metastatic Disease to the Pericardium mass centered in
[Figure 10]
the pericardium
• Hematogenous, lymphatic, and and associated with
contiguous routes a small pericardial
• Nodularity (tumor deposition), fibrinous effusion (arrows).
exudates, hemorrhagic effusion
• Lung, breast, lymphoma, melanoma,
renal, cardiac angiosarcoma
• Tamponade (15% of cases) Pericardium: The Essentials
• Pericardial sinuses and recesses mimic
pathology
Figure 10  Knowledge of normal anatomy, classic
Contrast- morphology and fluid density avoids
enhanced axial misdiagnosis
CT image from • A pericardial cyst has diagnostic imaging
a patient with
right atrial features
angiosarcoma  Cardiophrenic angle location, thin non-
shows multiple enhancing mesothelial capsule, simple
enhancing, fluid content
hypervascular • Pericarditis is the clinical expression of
nodules
(arrowheads) mesothelial inflammation with fibrinous
lining the exudates
pericardial sac  CT and MRI demonstrate pericardial
and a small effusion with smooth pericardial
high density
thickening and rapid contrast
(hemorrhagic) pericardial effusion compatible with
metastatic disease to the pericardium. enhancement
• Pericardial constriction is impaired
diastolic filling caused by non-compliant
adherent pericardium
Pericardium and Primary Malignancy  Conical ventricular deformity, dilated
• Lipoma
atria, pericardial thickening and
• Hemangioma
paradoxical septal “bounce” are
• Lymphangioma
supportive imaging findings
• Benign Teratoma
 Pericardial calcification may be present
• Paraganglioma
but is not diagnostic of constrictive
• Mesothelioma
physiology
• Sarcoma

The Pericardium Cardiac Imaging


1478
Pericardium: The Essentials
• Hemopericardium often accumulates
rapidly, increasing the risk of tamponade
 Pericardial effusion approaches soft
tissue density (>35–45 HU)
 Pericardial sac cannot accommodate
enlarging effusions without adequate
time to stretch and hypertrophy
• Metastatic disease to the pericardium
 May produce nodular pericardial
thickening and enhancement,
adhesions, effusion (occasionally
hemorrhagic) and even tamponade
 Lung cancer is the most common
primary
• Primary pericardial malignancy is rare
 Lipoma, hemangioma, lymphangioma,
benign teratoma, mesothelioma,
sarcoma and schwannoma are
reported

References
1. Chiles C, Woodard PK, Gutierrez FR, Link KM. Metastatic involvement of the heart and pericardium: CT
and MR imaging. RadioGraphics 2001;21:439–449.
2. O’Leary SM, et al. Imaging the pericardium: appearances on ECG-gated 64-detector row cardiac
computed tomography. The British Journal of Radiology 2010;83:194–205.
3. Broderick LS, Brooks GN, Kuhlman JE. Anatomic pitfalls of the heart and pericardium. RadioGraphics
2005;25:441–453.
4. Yared K, et al. Multimodality imaging of pericardial diseases. JACC: Cardiovascular Imaging 2010;3:650–
660.
5. Verhaert D, et al. The role of multimodality imaging in the management of pericardial disease. Circ
Cardiovasc Imaging 2010;3:333–343.
6. Napolitano G, Pressacco J, Paquet E. Imaging features of constrictive pericarditis: Beyond pericardial
thickening. Canadian Association of Radiologists Journal 2009;60:40–46.
7. Mambo NC. Diseases of the pericardium: Morphologic study of surgical specimens from 35 patients. Hum
Pathol 1981;12:978–987.
8. Peebles CR, Shambrook JS, Harden SP. Pericardial disease – anatomy and function. The British Journal of
Radiology 2011;84:S324–337.
9. Roberts WC. Pericardial heart disease: its morphologic features and its causes. Baylor University Medical
Center Proceedings 2005;18:38–55.
10. Costa IL, Bhalla S. Computed tomography and magnetic resonance imaging of the pericardium. Seminars
in Roentgenology 2008;234–245.
11. Oh KY, et al. Surgical pathology of the parietal pericardium: A study of 344 cases (1993–1999).
Cardiovascular Pathology 2001;10:157–168.
12. Grebenc ML et al. Primary cardiac and pericardial neoplasms: Radiologic-Pathologic correlation.
RadioGraphics 2000;20:1073–1103.
13. Wang ZJ, et al. CT and MR imaging of the pericardial disease. RadioGraphics 2003;23:S167–180.
14. Gilkeson RC, Chiles C. MR evaluation of cardiac and pericardial malignancy. Magn Reson Imaging Clin N
Am 2003;11:173–186.

Cardiac Imaging The Pericardium


1479
Aortic Disease

Aletta Ann Frazier, MD


Spectrum of Aortic Disease Aortic Aneurysm
• Aneurysm • Surgical indications
• Traumatic aortic injury  Ascending aortic caliber >5.5 cm
• Acute aortic syndromes (non-Marfan), >5.0 (Marfan)
• Aortitis  Descending aortic caliber >6.5 cm
• Neoplasia (non-Marfan), >6.0 cm (Marfan)
 Growth >1 cm/year
Supravalvular ascending aortic  Symptomatic
aneurysm [Figure 1] • Open repair versus endovascular repair
(EVAR)
• Complications
 Rupture (30%–68% risk without
surgery)
 Fistulous communication (esophagus,
airway)

Bicuspid Aortic Valve [Figure 2]

Figure 1 A, B & C

Axial CTA, three-


dimensional volumetric
reconstruction, and
oblique sagittal
maximum intensity
projection CT image. Figure 2 A,
B&C
Supravalvular ascending
thoracic aortic aneurysm
measuring almost 8 A 20-year-old
cm. Ascending aortic patient with
aneurysm of a 42-year- bicuspid aortic
old man with Marfan valve and
syndrome. ascending aortic
aneurysm. Axial
CT cuts show a
supravalvular
dilatation, 4.5
cm, with gradual
Aortic Aneurysm tapering to
• Ascending: cystic medial degeneration normal caliber
 Acquired and hereditary at the level of
 Bicuspid aortic valve (AV) the proximal
descending
 Marfan syndrome thoracic aorta.
• Descending: atherosclerosis Additional reconstructions obtained in multiple planes
focused on the aortic valve plane suggests a bicuspid
configuration, later confirmed at surgery.

Cardiac Imaging Aortic Disease


1481
A 34-year-Old Man status post Motor
Vehicle Accident [Figures 3 & 4] Figure 5

Figure 3 Enlarged
cardiac
A 34-year-old
silhouette,
man status post
abnormal
motor vehicle
superior,
accident. Portable
and right
supine view of
paramediastinal
the chest shows
contour.
an abnormal
Pulmonary
superior
edema pattern.
mediastinal
contour
suspicious for
vascular injury.
Left-sided rib
fracture noted.

Figure 6 A & B

Acute Aortic Syndromes – acute onset


chest or back pain…
• Acute aortic dissection
• Aortic intramural hematoma (IMH)
• Penetrating aortic ulcer (PAU)
• Ruptured aortic aneurysm
Figure 4 A, B & C
Acute Aortic Dissection
A 34-year-old man status post motor vehicle
accident. Axial lung windows confirm left-sided rib • Acute: <14 days from onset symptoms
fracture with associated pleural hemorrhage and • Intimal tear
probable underlying contusion. An early volumetric CT  Blood enters media (weakened by
shows focal bulblike contour along the medial aspect elastic degeneration, smooth muscle
of the distal aortic arch compatible with contained cell loss)
aortic tear: a posttraumatic aortic pseudoaneurysm
with adjacent periaortic hematoma.  High pressure flow creates “false”
lumen (propagates distal or proximal)
 Inward displacement of intima
 Aneurysmal dilatation of aorta
Traumatic Aortic Pseudoaneurysm • Atherosclerotic plaque (+/-)
• Blunt chest trauma/shearing force • Complications
• Aortic tear at aortic isthmus  Ischemia (mesenteric, cerebral)
 Fixed by ligamentum arteriosum  Rupture
 Contained by adventitia
• Focal saccular pseudoaneurysm Classification of Aortic Dissection
 Medial location • DeBakey
 Narrow neck  Type I: ascending + descending
 Intimal flap (+/-) thoracic aorta
• Hemorrhage (periaortic, mediastinal)  Type II: ascending aorta only
• Chronic in 2.5% (calcified rim)  Type IIIa: descending thoracic only
(distal to left subclavian)
A 52-year-Old Woman complains of  Type IIIb: descending thoracic +
increasing dyspnea with five pillow abdominal aorta
orthopnea [Figures 5 & 6]
• Stanford
• Three years ago, she fell off her front  Type A: ascending aorta (+/-
porch and was diagnosed with a descending)
perinephric hematoma  Type B: descending aorta only (distal
to left subclavian)

Aortic Disease Cardiac Imaging


1482
Imaging of Aortic Dissection
• CTA or MRI
• Intimal flap, often “spirals” longitudinally
• True lumen higher density (usually)
• Extent of dissection, +/- valvular
involvement
• Coronary and branch vessel involvement:
recons
• Hemorrhage (mediastinal, pericardial,
pleural)
 Fluid >30 Hounsfield units (HU)
• Ischemic complications
 Segmental hypoperfusion
 Pneumatosis, bowel wall thickening Figure 8 A & B
 Portal venous gas
Complex Stanford type A. Acute aortic dissection in a
Acute Aortic Dissection: Underlying 64-year-old man. Right brachio cephalic, left common
carotid, left subclavian all supplied by true lumen.
Causes Dissection flap extends to origin of left common
• 75% patients – chronic hypertension carotid and left subclavian vessels. Note, penetrating
(usually >70 years old) aortic ulcer along lateral margin of aortic arch. Actual
• Smoking – contributory extension of dissection – from aortic root to level of
renal arteries. Note, relative absence of atherosclerotic
• Intramural hematoma – predisposing disease.
• Bicuspid AV (patients <40 years old)
• Iatrogenic (5%)
• Connective tissue disorders
 Marfan syndrome
 Ehlers-Danlos syndrome
 Annuloaortic ectasia
• Pregnancy
• Cocaine use

A 64-year-Old Man with acute onset


chest pain [Figures 7 to 9]

Figure 9 A & B

Complex Stanford type A. Acute aortic dissection in a


64-year-old man. Patient underwent emergency repair:
open endovascular aortic repair with Dacron graft.
Specimen: ascending aorta – media wall separated
and expanded by accumulation of fibrin and clot
(almost 2 cm in thickness). No ulcerated or calcified
atherosclerotic plaques (remember: not a clear cause
of aortic dissection. Potentially “protective”).

Figure 7 A & B

Complex Stanford type A. Acute aortic dissection in


a 64-year-old man. Dissection extending from sino-
tubular junction (ascending aorta) to aortic bifurcation.
Aneurysmal dilatation of ascending aorta (>5.5 cm).

Cardiac Imaging Aortic Disease


1483
A 66-year-Old Man with Chest Pain
[Figure 10]

Figure 12 A & B

Coronal maximum intensity projection again shows


aneurysmal dilatation and dissection of ascending
thoracic aorta, with dissection flap extending into right
brachiocephalic artery. Sagittal maximum intensity
projection shows the endograft evident in descending
aorta, with evidence of a second dissection flap
originating near the level of the hiatus.

Figure 10 A to E

Axial CT images and coronal maximum intensity


projection reformats show aneurysmal dilatation of
the ascending thoracic aorta (measured greater than
5 cm). Dissection flap originating at the root of the
aorta, extending to (and involving) takeoff of the
great vessels. Although left coronary appears off true
lumen, but right coronary was not well-visualized due
to cardiac motion. No hemopericardium.

A 72-year-Old Woman with history of


endograft placement for descending
aortic aneurysm with vague chest
discomfort [Figures 11 to 13]
Figure 13 A,
B&C

Coronal
maximum
intensity
projection
again shows
aneurysmal
dilatation and
dissection
of ascending thoracic aorta, with dissection flap
Figure 11 A, B & C extending into right brachiocephalic artery.
Axial CT at apex and coronal maximum intensity
Selected axial CT cuts projections of the neck. Dissection flap extends into
show aneurysmal origin of right brachiocephalic vein and left common
dilatation and carotid arteries with marked proximal luminal
dissection of ascending narrowing (slitlike origin of left common carotid, with
thoracic aorta, with large amount eccentric thrombus). Bilateral common
crescentic intramural carotid arteries patent although left common carotid
or periaortic thrombus. artery narrowing is evident.
Left and right coronary
arteries fed by true
lumen. Small right
effusion 30 HU,
suspicious for pleural
hemorrhage. Endograft evident in descending aorta.
Note, fingerlike projection of false lumen into lateral
wall, right PA – an area of usual thinning with risk of
adjacent PA dissection/rupture?

Aortic Disease Cardiac Imaging


1484
A 68-year-Old Man with severe chest
and abdominal pain [Figures 14 to 16]

Figure 16 A,
B&C
Figure 14 A, B & C
Abdominal
Selected multi- aorta containing
detector CT images hematoma in
demonstrate false lumen.
aneurysmal dilatation
of ascending aorta
with large type A aortic
dissection, extending
from the root of the
aorta.The right and
left coronaries are
patent, but there
is a high density
pericardium effusion
(45 HU) compared with
hemopericardium.
Dissection propagates
into descending
thoracic aorta common carotid, with large amount A 64-year-Old Man, enlarging left-
eccentric thrombus). Bilateral common carotid arteries sided aortic arch contour over past 12
patent although left common carotid artery narrowing is months [Figures 17 to 19]
evident.

Figure 17

A 64-year-old man with slowly enlarging type B


Figure 15 A to E dissection (identified over the past 12 months).
Selected multi-detector CT images demonstrate
aneurysmal dilatation of ascending aorta with large
type A aortic dissection, extending from the root of
the aorta. The right and left coronaries are patent,
but there is a high density pericardium effusion (45
HU) compared with hemopericardium. Dissection
propagates into descending thoracic aorta and
abdomen (celiac, superior mesenteric artery, left
renal off true lumen; right renal and aortic intramural
hematoma off false lumen). Dissection involves right
common iliac artery.
Cardiac Imaging Aortic Disease
1485
A 72-year-Old Woman with chronic
hypertension and acute onset of back
pain [Figures 20 to 22]

Figure 20 A to F
A 72-year-old woman with chronic hypertension,
acute onset back pain. Selected axial cuts. Non-
contrast matched to post-contrast CT images. Gross:
Figure 18 A, B, C & D extensive medial hematoma, no intimal tear. Type B
aortic intramural hematoma originated distal to left
Type B aortic dissection with 2 entry sites.
subclavian, propagated to level of superior mesenteric
One intimal flap evident in the aortic arch, just distal
artery.
to take-off of left subclavian artery. True lumen is
smaller than the false, and is compressed against the
right side of the descending thoracic aorta. Flow in
both false and true lumens within descending thoracic
aorta, which show a second intima flap at the level of
the left main pulmonary artery. Bottom Right Image:
shows continued distal propagation of dissection, which
extended well below the diaphragm into the internal
and external iliac arteries.

Figure 21 A & B
A 72-year-old woman with chronic hypertension, acute
onset back pain. Treated medically and discharged. The
patient returns 4 days later with severe back and throat
pain. Proximal extension to ascending aorta (type A
aortic intramural hematoma). Gross: extensive medial
hematoma, no intimal tear. Type B aortic intramural
hematoma within 4 days demonstrated proximal
Figure 19 A, B & C
extension to ascending aorta (type A).
Three-dimensional volumetric reconstructed images
and sagittal maximum intensity projection confirm
Type B aortic dissection. There is flow on both true
and false lumens.

Aortic Disease Cardiac Imaging


1486
A 56-year-Old Man with Acute back
pain [Figure 23]

Figure 22 A & B
Gross: transverse slice of ascending aorta shows fresh
thrombus within medial layer and bluish discoloration
of aortic wall (aortic wall ischemia, infarction). No
intimal tear identified. Type B aortic intramural
hematoma within 4 days demonstrated proximal
extension to ascending aorta (type A).

Intramural Hematoma
• Rupture of vaso vasorum
 Medial hemorrhage
 Enlarged aortic diameter
 No intimal tear (controversial) Figure 23 A, B, C & D
 60% arise in descending aorta A 56-year-old man with acute chest/back pain.
 >90% in descending aorta associated
with PAU
• 35% rupture
• 33% lead to aortic dissection A 63-year-Old Woman with acute onset
• Ascending IMH requires surgical repair
of severe back pain [Figure 24]

Imaging of Intramural Hematoma


• Noncontrast CT
 Peripheral crescent-shaped high
density
 Displaced intimal calcifications
• Contrast-enhanced CT
 Crescentic area low density and
unenhanced (vs false lumen of classic
dissection, which partly enhances
unless thrombosed)
 Constant, fixed circumferential density
(versus classic dissection which often
spirals longitudinally around aorta)
• Resorption or progression, stability or
migration
 IMH is a dynamic process
 Change often observed in first 30 days

Figure 24 A, B, C & D
A 63-year-old woman with acute onset intense back
pain. Bottom Images: postendograft for penetrating
aortic ulcer.

Cardiac Imaging Aortic Disease


1487
Penetrating Aortic Ulcer … A partly thrombosed aortic arch
• Atheromatous plaque focally ulcerates pseudoaneurysm – associated with
 Tunnels into media extensive ulcerated atherosclerotic
 Adjacent intramural hematoma (+/-) plaque.
• 90% arise in descending aorta
• Complications Pseudoaneurysm is a well-recognized
 Pseudoaneurysm complication of penetrating aortic
 Dissection
 Rupture
ulcer, and the lesion May have begun
• Measurements for repair like this … [Figure 27]

 Width >2 cm
 Depth >1 cm

A 74-year-Old Woman with swallowing


difficulty [Figures 25 & 26]

Figure 27 A & B
A penetrating aortic ulcer.

Figure 25 A & B A 56-year-Old Woman, steroid-


A 74-year-old woman with saccular aneurysm at level dependent status post lung transplant
of transverse aorta. with septic arthritis, renal abscesses,
and Cytomegalovirus pneumonia.
She has disseminated Scedosporium
prolificans fungal infection [Figure 28]

Figure 28 A, B & C
The patient expired and autopsy revealed severe,
necrotizing fungal aortitis. Cut section of the affected
abdominal aorta shows hyperplastic intima and
necrotic edematous media/adventitial layers.
Figure 26 A to E Viable, dividing fungal forms involving a 6-cm length
of the infrarenal abdominal aorta was noted at
A 74-year-old woman with saccular aneurysm at level autopsy examination.
of transverse aorta and diffuse penetrating aortic
ulcer. Axial CT images with surface rendered images.
Partially thrombosed aortic arch aneurysm,
aneurysmal dilatation of common origin of innominate
and left carotid artery, extensive partly ulcerated
atherosclerotic plaque.

Aortic Disease Cardiac Imaging


1488
Aortitis
• Noninfectious (isolated vs systemic
autoimmune)
 Takayasu arteritis
 Giant cell aortitis
 Rheumatologic diseases
 Other systemic autoimmune diseases
(e.g., Crohn, lupus)
 Nonspecific granulomatous aortitis
• Infectious (less common)
 Bacterial (Salmonella, Staphylococcus,
Streptococcus)
 Fungal
 Tuberculous
 Syphilitic

Aortitis [Figure 29]


• Medial necrosis  inflammation  fibrosis
with intimal and adventitial scarring
• Predilection for ascending aorta
 Occasionally “skip” areas
 Intima thickened, surface “wrinkled”
• Complications
 Aneurysm, rupture
 Stenoses (especially aortic branches)

Figure 29 Figure 30 A to E

Aortitis is A 58-year-old man


characterized with recurrent
by chest pain. IV
inflammation contrast reveals
of the aortic extensive mural/
wall, which intimal thrombus
may be acute which dramatically
or chronic (and narrows the lumen
partly fibrotic). of the descending
thoracic aorta.
Outer contour of
aorta also appears
somewhat irregular
but no definite
penetrating aortic
Imaging in Aortitis ulcer or aortic intramural hematoma. Abnormality
• Mural thickening clears just above level of crus. Aortic sarcoma
• Luminal thrombosis mimicking extensive intimal thrombus.
• Aortic aneurysm
• Mural edema (on MRA) Aortic Sarcoma
• Stenosis of branch vessels • Intimal neoplasia, mixed with adjacent
• Calcification (chronic) organized thrombus
• Mimics severe atherosclerotic disease or
A 58-year-Old Man with recurrent intimal thrombus
chest pain over the past three months • Poor survival (8% at 5 years)
[Figure 30]  Branch stenosis/occlusion/ischemia
 Local infiltration/recurrence
 Distant metastases or embolization
 Aortic rupture due to tumor necrosis
• Treatment
 Endarterectomy vs resection/graft
interposition
 Adjuvant chemo

Cardiac Imaging Aortic Disease


1489
A 74-year-Old Woman with acute Key Information to Report – Aortic
onset renal failure and intermittent Dissection
abdominal pain [Figures 31 & 32] • Stanford A versus B
• Site of intimal flap origin
• Discernment true vs false lumen
• Extension into aortic valve, coronary ostia
• Extension into aortic arch branches
• Diameter of aorta (ascending, descending)
• Evidence of leakage or rupture
• Hemopericardium
• Evidence of organ or bowel ischemia

Key Information to Report – Intramural


Figure 31 A, B & C Hematoma
Aortic sarcoma in a 74-year-old woman with • Site of IMH
intermittent abdominal and back pain (abdominal • Regional extent of IMH
angina) and new onset renal failure. Sagittal (black • Evidence of etiology (PAU, trauma)
blood) MR images show large, partly lobulated, wide- • Site(s) of aneurysmal dilatation
based filling defect in the mid abdominal aorta at the
level of the kidneys. • Evidence of leakage or rupture

Key Information to Report – Penetrating


Aortic Ulcer
• Location of ulcer
• Multiplicity of ulcers
• Evidence of IMH
• Sites of saccular aneurysm or
pseudoaneurysm
• Evidence of leakage or rupture
• Relationship to left subclavian, celiac,
renal arteries (for potential endovascular
stenting)

Figure 32 A, B & C
Postgadolinium coronal (bright blood) MR images
show a large mural-based anterolateral filling defect
causing localized aortic stenosis, occlusion of right
renal artery, stenosis superior mesenteric artery and
celiac artery. Gross: 6-cm segment of abdominal
aorta shows polypoid mass associated with calcified
atheromatous plaque occluding right renal artery.

Spectrum of Aortic Disease


• Aneurysm
 Size, growth rate criteria for impending
rupture
• Traumatic aortic injury
 Isthmus location, acute vs chronic
features
• Acute aortic syndromes
 Painful onset of dissection, IMH, PAU:
dynamic lesions
• Aortitis
 Usually autoimmune, may progress
to aneurysm or aortic fibrosis/branch
stenosis
• Neoplasia
 Intimal thrombus admixed with
malignant cells, often partly occlusive

Aortic Disease Cardiac Imaging


1490
Ischemic Cardiomyopathy

David A. Bluemke, MD, PhD, FACR


Cardiomyopathy 2 hours after experimental infarction
• Ischemic [Figure 2]
 Coronary artery disease
• Nonischemic
 Myriad of causes, “primary” muscle
disorder

Edema in acute Myocardial Infarction


[Figure 1]

Figure 1A

T2 (edema)
Figure 2 A to F

Canine model of reperfused and not-reperfused infarct


= hyperenhancement correlates well with nonviable
tissue, comparing 1 day, 3 days, and 8 weeks after
myocardial infarction. No recovery was seen in areas
of transmural delay enhancement.

Q-wave infarction [Figure 3]

Figure 1B

Gadolinium
T1
(necrosis +
scar)
M.Friedrich

Figure 3 A, B & C

Courtesy of M. Friedrich Q-wave infarction.

Coronary Heart Disease: Acute coronary


blockage (infarct)
• Initially, there is myocardial necrosis (cell
death) that also enhances with gadolinium
• If the blockage is not rapidly opened and
the patient survives, the capillary blood
supply may also be damaged

Microvascular Obstruction [Figure 4]


• Microvascular obstruction (MO)
predicts significantly increased rate of
cardiovascular complications after MI
(unstable angina, reinfarction, heart
failure (HF), embolic stroke, death)

Wu KC, et al. Circulation 1998;97:765-772.

Cardiac Imaging Ischemic Cardiomyopathy


1491
Subacute infarction [Figure 7]
• Right coronary artery (RCA) infarct:
inferior wall left ventricle and right
ventricle

Figure 4

Microvascular
obstruction.

1st pass perfusion abnormality [Figure 5]


Figure 7 A to H
Subacute infarction.

Chronic infarct: 6 months, remodeling


[Figure 8]

Figure 8

Chronic
infarct.

Figure 5
1st pass perfusion abnormality.

Acute myocardial infarction: subacute Chronic infarct: 6 months, clot [Figure 9]


complications
• Myocardial rupture: tamponade
• Papillary muscle rupture
• Myocardial remodeling
• (Myo)pericarditis (Dressler syndrome)

Impending myocardial rupture [Figure 6]

Figure 6 A & B

Acute myocardial infarction.

Figure 9 A to I
Chronic infart.

Ischemic Cardiomyopathy Cardiac Imaging


1492
Chronic myocardial infarction

72-year-Old Woman with congestive


heart failure, known coronary artery
disease: MRI cine and late gadolinium
enhancement images [Figure 10]
Figure 12

Hibernating
myocardium.

Enhancement of myocardium versus


myocardial scar [Figure 13]
Figure 10 A & B • Images every 1 minute after gadolinium
Woman with massive myocardial infarction. injection, chronic myocardial infarction
M. Friedrich, AHA. 2002.

Nonviable Myocardium – Cell death of


myocytes Figure 13
• Clinical tests:
Canine model of
 a) Response to inotropic stimulation
reperfused and
(dobutamine echo) not-reperfused
 b) Presence of glucose metabolism infarct = hyper-
(PET) enhancement
 c) Presence of cellular transport correlates
well with non-
mechanisms (Tl-201 SPECT) viable tissue,
 d) Presence of fibrosis (MRI or CT) comparing 1
day, 3 days,
Stunned Myocardium [Figure 11] and 8 weeks
• Acute MI after myocardial
 Decreased function infarction. No
recovery was
 Normal blood flow seen in areas of transmural delay enhancement.
 Has the potential to recover function
 No fibrosis on MRI
Reports [Figure 14]
• Identify segments
• Transmural or subendocardial
• Function (EF, SV, EDV, ESV)

Figure 11

Stunned
myocardium.

Figure 14 A, B, C & D

Reports.
Hibernating Myocardium [Figure 12]
• Reduced contraction at rest +
• Chronically reduced blood flow
• Function can improve after
revascularization
• Flow limiting coronary artery disease
(CAD) but no infarct

Cardiac Imaging Ischemic Cardiomyopathy


1493
65-year-Old Woman, acute onset Heart Conclusion
Failure [Figure 15] • Enhancing areas on delayed gadolinium
MRI or CT represents scar or tissue
necrosis
• Acute MI: perfusion defect on first
pass images represents microvascular
obstruction

Figure 15 A & B
Left Image: Short axis SSFP.
Right Image: Two-chamber SSFP.

Clot formation in Right Coronary Artery


Aneurysm – delayed Gadolinium most
sensitive sequence [Figure 16]

Figure 16 A to K

Hyperenhancing myocardium extending from the


anterior wall at the base. It became circumstantially at
the apex. Smooth, regular mural thrombus are well-
demonstrated from the anterior wall at the base with
a circumferential distribution at the apex. Areas of
transmural infarction in the septum and anterior wall.

Ischemic Cardiomyopathy Cardiac Imaging


1494
Cardiomyopathies, Nonischemic

Vincent B. Ho, MD, MBA


Cardiomyopathies Primary Cardiomyopathies (American
• Old definition (World Health Heart Association 2006)
Organization)* • Genetic
 1968: “diseases of different and  Hypertrophic
often unknown etiology in which the  Arrhythmogenic right ventricular
dominant feature is cardiomegaly and dysplasia/cardiomyopathy (ARVD/C)
heart failure”  Left ventricle (LV) noncompaction
 1980: “heart muscle diseases of  Glycogen storage
unknown cause”  Mitochondrial myopathies
 1995: “diseases of myocardium  Ion channel disorders
associated with cardiac dysfunction” • Mixed
• Old classification  Dilated
 Morphology (hypertrophic, dilated)  Restrictive (nonhypertrophied and
and function (restrictive) nondilated)
 Problem: • Acquired
™™ sarcomeric gene mutations:  Inflammatory (myocarditis)
dialated cardiomyopathy (DCM)  Stress-provoked (takotsubo)
and hypertrophic cardiomyopathy  Peripartum
(HCM)  Tachycardia-induced
™™ Troponin I mutations HCM and  Infants of insulin-dependent diabetic
restrictive cardiomyopathy (RCM) mothers
*Maron, et al. Circulation. 2006.
Imaging of Cardiomyopathies
American Heart Association Defintion • Function
• “Cardiomyopathies are a heterogeneous  Ejection fraction (EF)
group of diseases of the myocardium  Wall motion
associated with mechanical and/or • Myocardial characterization
electrical dysfunction that usually (but  Ischemic vs nonischemic process
not invariably) exhibit inappropriate  Acute vs chronic
ventricular hypertrophy or dilatation • MRI typically best (T2-weighted,
and are due to a variety of causes that myocardial delayed enhancement [MDE])
frequently are genetic.” • Multidetector CT (MDCT) for select
• “Cardiomyopathy” excludes myocardial patients (e.g., implantable cardioverter
pathologies that result from other defibrillators [ICDs])
cardiovascular abnormalities such as:
Myocardial Delayed Enhancement
 Valvular heart disease
[Figure 1]
 Systemic hypertension
• 10–20 minute post gadolinium (Gd) (0.2
 Congenital heart disease
mmol/kg)
 Atherosclerotic coronary artery
• Inversion time (TI) set to null normal
disease
enhancing myocardium
 Metastatic disease
• Hyperenhancement is ABNORMAL
 Cardiac tumors
• Myocardial infarction
• “Cardiomyopathies either are confined
 “Hyperenhancement” on MDE
to the heart or are part of generalized
 Subendocardial involvement
systemic disorders, often leading to
cardiovascular death or progressive heart Kim RJ, et al. N Engl J Med. 2000;343:1445.
failure-related disability.” Choi KM, et al. Circulation. 2001;104:1101.
• Classification
 Primary (genetic, mixed, acquired)
 Secondary (multiorgan)
*Maron, et al. Circulation. 2006.

Cardiac Imaging Cardiomyopathies, Nonischemic


1495
Figure 1 Hypertrophic Cardiomyopathy
• Hypertrophied, nondilated LV in absence
Myocardial delayed
of another systemic or cardiac disease
enhancement
imaging of a (e.g., hypertension [HTN] or aortic
patient with stenosis)
myocardial • Histology: myocardial disarray and fibrosis
infarction. • 1:500 incidence: in US, most common
On short axis
myocardial
cause of sudden cardiac death (SCD) in
delayed young and important substrate for heart
enhancement failure at any age
images, a • Autosomal dominant – 11 mutant genes
myocardial identified (e.g., ß-myosin heavy chain)
infarction of
the lateral encoding contractile proteins of the
wall of the left cardiac sarcomere
ventricle is seen • Presentation: chest pain, syncope, SCD
as subendocardial hyperenhancement on Myocardial
delayed enhancement (arrows). Image courtesy of Hypertrophic Cardiomyopathy
Thomas Foo, Ph.D.
[Figure 3]
• Morbidity and mortality:
Myocardial Delayed Enhancement:  Sudden cardiac death (ventricular
Hyperhencement Patterns [Figure 2] tachycardia)
 Obstruction (surgery or ablation)
• Imaging:
 Myocardial thickening (diastolic LV wall
≥15 mm)
 Location: focal (especially anterior,
anteroseptal), concentric, and apical
 Function: obstruction (especially left
ventricle outflow tract) or systolic
anterior motion of mitral valve leaflet?

Figure 2
On myocardial delayed enhancement imaging,
abnormal regions can be identified as regions of
hyperenhancement—regions with brighter enhancement
than that of normal myocardium. In patients
with ischemic heart disease, myocardial infarction
typically affects the end-vessel territory which is the
subendocardial (inner) layer of myocardium but if
more extensive can extend outward to involve the
entire myocardial wall (i.e., full thickness). Patients
with greater than 50% wall hyperenhancement are
much less likely to benefit from a revascularization
procedure. The cardiomyopathies comprise the various
non-ischemic myocardial diseases. In patients with
hypertrophic cardiomyopathy, the myocardium is
thickened and areas of fibrosis if present can be seen
on myocardial delayed enhancement as patchy mid-
myocardial hyperenhancement. Patients with dilated
cardiomyopathy often have curvilinear mid-myocardial
hyperenhancement in association with a dilated
ventricle. Patients with myocarditis often have a sub-
epicardial distribution of hyperenhancement which
typically is in the inferolateral wall but may involve
other myocardial segments. Patients with cardiac
sarcoidosis may have a variety of appearances ranging
from patchy and confluent to more nodular; but typically
cardiac involvement is a late feature and the patient
typically has a known history of sarcoidosis in whom
mediastinal and/or hilar adenopathy is present. Cardiac
amyloidosis typically presents with extensive sub-
endocardial hyperenhancement that is diffuse and often
circumferential typically with a dark blood pool that
makes proper selection of inversion time for myocardial
delayed enhancement difficult.

Cardiomyopathies, Nonischemic Cardiac Imaging


1496
Arrhythmogenic Right Ventricular
Dysplasia/Cardiomyopathy
• Progressive cardiomyopathy characterized
by ventricular arrhythmias that may lead
to SCD
• Histology: Loss of myocytes with fatty/
fibrofatty replacement of the right
ventricle… but may involve the left
ventricle
• ~1:5,000; males > females; 20–40 year
olds (US: 5% SCD in <35 years olds;
especially Italian athletes)
• Familial (30%–50%; autosomal dominant;
8 loci in 4 genes, e.g., desmoplakin and
plakophillin-2)
• Palpitations, syncope, sudden death

Arrhythmogenic Right Ventricular


Dysplasia/Cardiomyopathy
• Not an easy diagnosis even by MRI
Figure 3 A, B, C & D
• In ARVD patients, fibrofatty infiltration
Hypertrophic cardiomyopathy. On short axis steady- may be difficult to see using MRI
state free precession imaging (A), characteristic • Right ventricle (RV) fat can be seen in
thickening of the septal wall (arrow) of the left ventricle patients without ARVD. Examples: aging,
is identified. On the end-systolic short axis myocardial obesity, steroid therapy, …lipomatous
tagging image (B), persistence of the square myocardial
tagging grids (yellow circle) is seen in the hypokinetic
infiltration of RV (Macedo. AJR. 2007.)
thickened septal wall [compare with the significant • Fatty infiltration is less specific and least
deformation of the myocardial tagging grids in the normal reproducible MR parameter for ARVD
contracting inferolateral wall (green circle)]. On short compared to other RV features, notably
axis perfusion imaging (C), regions of lower perfusion regional RV dysfunction (Tandri. JACC.
are noted in the mid to subepicardial region of the
thickened anteroseptal wall of the left ventricle (arrow). 2006.; Bluemke. Cardiology. 2003.)
On short axis myocardial delayed enhancement imaging • Based on major and minor criteria
(D), the anteroseptal region of diminished perfusion proposed by the Task Force on
is noted to be hyperenhanced on myocardial delayed Cardiomyopathy in 1994 (*McKenna, et
enhancement (large arrow). Additional patchy regions al. Br Heart J. 1994.)
of hyperenhancement are noted in the inferoseptal wall
of the left ventricle (small arrow) in the mid-myocardial • 6 categories of major and minor criteria
region. based on various electrocardiogram,
pathophysiologic, histopathologic, and
imaging abnormalities
• Diagnosis of ARVD:
Hypertrophic Cardiomyopathy  2 major criteria,
• Risk assessment by MDE  1 major/2 minor criteria, or
 Hyperenhancement correlates with  4 minor criteria
increased risk factors for SCD (Moon.
JACC. 2003.)
 Hyperenhancement correlates with
fibrosis vs myocardial disarray (Moon.
JACC. 2004.)
 7-fold increase risk for nonsustained
ventricular tachycardia on Holter (28%
vs 4%; Adabag. JACC. 2008.)
 Fibrosis on MDE independent predictor
of adverse outcome (heart failure,
24.5% vs 9.9%; arrhythmic outcome,
7.3% vs 2.5%; O’Hanlon. JACC.
2010.)

Cardiac Imaging Cardiomyopathies, Nonischemic


1497
Global and/or Regional dysfunction and Left Ventricular Noncompaction
Structural Alterations [Figure 5]
• Major
 Localized RV aneurysms
 Severe RV dilation and reduced RV EF Figure 5
with no/mild LV impairment
 Severe segmental dilation of RV Left ventricular
noncompaction.
• Minor Increased
 Mild global RV dilation and/or reduced trabeculation
RV EF with normal LV (arrows) is the
 Mild segmental dilation of the RV typical imaging
 Regional RV hypokinesis feature seen in
left ventricular
noncompaction.
Arrhythmogenic Right Ventricular Image courtesy
Dysplasia/Cardiomyopathy [Figure 4] of Jean Jeudy,
MD.

Figure 4

Arrhythmogenic Dilated Cardiomyopathy


right ventricular • Dilated ventricular chamber characterized
dysplasia. by systolic dysfunction with normal LV
On systolic wall thickness
4-chamber cine
steady-state • Genetic (20%–35%; especially autosomal
free precession dominant) and nongenetic forms (often
image, “idiopathic”)
characteristic • ~1:2,500 incidence; third most common
microaneurysms
cause of heart failure and most common
(focal dyskinetic
regions) are indication for heart transplant
seen in the • Typically 30–50 years old but can be seen
free wall of the in children
right ventricle. This “crinkling” of the right ventricular • Presentation: systolic dysfunction, heart
free wall also called the “accordion sign,” was described
by Dalal D, et al. in JACC 2009. Also reported was a
failure, shortness of breath, peripheral
very high association of this feature with patients with edema, thromboemboli, arrhythmias,
desmosomal mutations associated with arrhythmogenic sudden cardiac death
right ventricular dysplasia. Image courtesy of Jean
Jeudy, MD. Imaging of Dilated Cardiomyopathy
• Distribution
 Global
• MRI
Left Ventricular Noncompaction  Diminished LV ejection fraction
• Deep “spongy” appearance to the left  Maintenance of normal myocardial
ventricular wall that results from an end-diastolic wall thickness
arrest in normal embryogenesis of the  Hyperenhancement on MDE in the
myocardium mid-myocardial (mesocardial zone vs
• Isolated feature or associated with subendocardial as seen in myocardial
congential heart disease (CHD) (complex infarction)
cyanotic CHD)
• Familial and nonfamilial forms; also,
genetic mutations associated with CHD
(α-dystrobrevin gene and transcription
factor NKX2.5)
• Presentation: systolic dysfunction/heart
failure (HF), thromboemboli, arrhythmias,
sudden cardiac death

Imaging of Left Ventricle


Noncompaction
• Distribution
 Apex
• MRI
 Diminished LV ejection fraction
 Increased LV trabeculation
 May see hyperenhancement on MDE

Cardiomyopathies, Nonischemic Cardiac Imaging


1498
Dilated Cardiomyopathy [Figure 6] Myocarditis [Figure 7]

Figure 6 A & B

Dilated cardiomyopathy. Adult man with heart Figure 7 A, B & C


failure is noted on cine steady-state free precession
images (A, end-diastolic short axis steady-state free Myocarditis in an
precession image) to have a dilated left ventricle (end- adult man with a
diastolic dimension is 75 mm) and a left ventricular 2-day history of
ejection fraction of 19%. On short axis myocardial dull, nonpleuritic
delayed enhancement imaging (B), curvilinear chest pain and
hyperenhancement (arrow) is noted most prominently found to have
in the mid-myocardial region of the septal wall of the elevated serum
left ventricle. troponin levels.
On short-axis
steady-state
free precession
imaging, there is
Myocarditis subtle curvilinear
• Typically infection (viral: Coxsackie, subepicardial
cytomegalovirus, human region of increased
immunodeficiency virus; bacterial; signal in the
inferolateral wall
fungal); other causes (e.g., postvaccine) of the left ventricle
• Histology: inflammation and edema (arrow in A). On T2-weighted imaging, increase
of myocardium (and occasionally signal is also noted in the inferolateral wall (arrow
pericardium) in B). Corresponding short axis myocardial delayed
enhancement image also shows hyperenhancement in
• Presentation: typically acute, chest pain,
the inferolateral wall of the left ventricle. In addition
fever, malaise/fatigue, shortness of to the patient’s history, the presence of edema and an
breath, heart failure, arrhythmia acute myocardial process is suggested by the presence
• Prognosis: typically resolves but may also of increased signal on T2-weighted as well as steady-
lead to permanent HF state free precession imaging.

Imaging of Myocarditis
• Distribution
 Typically subepicardial location
Secondary Cardiomyopathies (American
(especially inferolateral) Heart Association 2006)
 May involve the pericardium • Infiltrative (amyloidosis, Gaucher, Hurler,
(myopericarditis) Hunter)
• MRI • Storage (hemochromatosis, Fabry,
 Diminished LV ejection fraction Niemann-Pick)
 Increased T2 signal (edema) • Toxicity (drugs, heavy metals, chemicals)
 Hyperenhancement on MDE • Endomyocardial (endomyocardial fibrosis,
 Typically resolves over time (vs other Loeffler endocarditis)
cardiomyopathies) • Inflammatory (sarcoidosis)
• Endocrine (diabetes, hyperthyroidism,
hypothyroidism, hyperparathyroidism,
pheochromocytoma)

Cardiac Imaging Cardiomyopathies, Nonischemic


1499
Secondary Cardiomyopathies (American Cardiac amyloidosis [Figure 8]
Heart Association 2006)
• Neuromuscular (NF, TS, Friedreich ataxia,
Duchenne-Becker, myotonic dystrophy) Figure 8
• Cardiofacial (Noonan syndrome) Cardiac
• Nutritional (beriberi, pellagra, scurvy, amyloidosis.
selenium, carnitine, kwashiorkor) On myocardial
• Autoimmune (systemic lupus delayed
enhancement
erythematosus, dermatomyositis, imaging,
rheumatoid arthritis, scleroderma, subendocardial
polyarteritis nodosa) hyper-
• Electrolyte imbalance enhancement
• Consequence of cancer therapy (arrows), which
is typically
(anthracyclines, cyclophosphamide, diffuse and
radiation) circumferential,
is characteristic
Amyloidosis for cardiac amyloidosis. In addition, there is often a
• Amyloid (insoluble proteins; paucity of signal in the blood pool which makes proper
selection of an appropriate inversion time difficult for
immunoglobulin light chain, [AL]
myocardial delayed enhancement.
amyloidosis; and transthyretin, [TTR])
infiltration and expansion of interstitial
space associated with endomyocardial
fibrosis; pink on Congo red staining
• Multi-organ: kidney, heart, Sarcoidosis
gastrointestinal, liver, lungs • Myocardial involvement by noncaseating
• Disease is progressive; cardiac granulomas containing epithelioid cells
involvement cause of death in 50% of and large multinucleated giant cells
patients with AL amyloidosis • Sarcoidosis: 1–64/100,000 worldwide
• Presentation: diastolic heart failure • Cardiac sarcoidosis: 4%–5% of patients
(restrictive physiology), peripheral edema, (premortal) vs 20%–25% (autopsy);
arrhythmias, chest pain, death Japan: 58% patients and cardiac sarcoid
responsible for 85% patients’ deaths
Imaging of Cardiac Amyloidosis • Presentation: typically with known sarcoid
• Distribution history (but not always), arrhythmia,
 Subendocardial heart block, SCD
 Diffuse
• MRI Imaging of Cardiac Sarcoidosis
 Left ventricular wall thickening • Distribution
 Poor diastolic function  Highly variable
 Typically difficult inversion time  Does not follow distinctly vascular
selection for MDE imaging… poor pattern
ventricular signal (related to systemic • MRI
amyloid “load”). Maceira. Circulation.  Hyperenhancement on MDE
2005. ™™ Patchy
 Diffuse subendocardial pattern of ™™ Nodular
hyperenhancement ™™ Mixed
 May mimic other cardiomyopathies
 Typically patient has history or
evidence for sarcoidosis (mediastinal
and/or pulmonary disease)

Cardiomyopathies, Nonischemic Cardiac Imaging


1500
Cardiac Sarcoidosis [Figure 9]

Figure 9 A, B, C & D

Cardiac sarcoidosis. In cardiac sarcoidosis, non-


caseating granulomas can be seen on MRI. In this
adult patient with a known history of sarcoidosis and
an arrhythmia, short-axis T2-weighted imaging (A)
reveals several nodular regions of increased signal in
the right ventricular outflow tract (yellow arrows) and
in the left ventricular inferoseptal wall (green arrow).
On short axis myocardial delayed enhancement
imaging (B), these nodular areas are hyperenhancing
(yellow arrows, right ventricular outflow tract;
green arrow, inferoseptal wall of left ventricle). On
3-chamber myocardial delayed enhancement imaging
(C; D, 3-chamber steady-state free precession
image for anatomic correlation), scattered nodular
regions of hyperenhancement are noted in the right
ventricular free wall (C, yellow arrow) as well as the
interventricular septum (C, green arrows).

Summary
• Cardiomyopathies
 Primary (genetic, mixed, acquired)
 Secondary
• Morbidity/mortality
 Mechanical (cardiac function, heart
failure)
 Electrical (arrhythmias, SCD)
• MRI best for assessment
 T2-weighted imaging (acute vs
chronic)
 MDE (ischemic vs nonischemic
process)

Cardiac Imaging Cardiomyopathies, Nonischemic


1501
Coronary Artery Anatomy and Pathology

Seth J. Kligerman, MD
Coronary Arteries Figure 2
• Epicardial vessels
Left main
• Provide oxygenated blood to myocardium coronary
• Divisions artery (yellow
 Left main arrow) which
™™ Left anterior descending (LAD) arises from
left coronary
™™ Left circumflex (LCx)
cusp (star) and
 Right divides into
left anterior
Aortic Valve [Figure 1] descending
• Three sinuses coronary
 Anatomic dilation of ascending aorta artery (LAD,
red arrow).
 Just above valve Left circumflex
• Three cusps coronary artery
 Right coronary cusp (RCC) (LCx, blue
™™ Points anteriorly arrow).
™™ Right coronary artery (RCA) origin
 Left coronary cusp (LCC)
Left Main Anatomy Variants [Figure 3]
™™ Points to left
™™ Left main origin
 Noncoronary cusp (NCC)
™™ Points between atria
• Commissures are junction between
sinuses

Figure 1
Figure 3 A & B
Aortic valve.
Right
coronary Left Image: Ramus intermedius – 30%.
cusp (RCC), Right Image: Separate ostium – 2%.
noncoronary
cusp, and
left coronary
cusp (LCC). Left anterior descending Anatomy
[Figure 4]
• LAD supplies anterior surface of heart
• Lateral branches are diagonals
• Medial branches are septal branches
Left Main Anatomy [Figure 2] • LAD divided into proximal, mid, and distal
• Left main coronary artery segments by septals
 Arises from left coronary cusp (star)
 Divides into
™™ Left anterior descending coronary
artery (LAD, red arrow)
™™ Left circumflex coronary artery
(LCx, blue arrow)

Figure 4 A & B
Left anterior descending (arrows) anatomy.

Cardiac Imaging Coronary Artery Anatomy and Pathology


1503
Left Circumflex Anatomy [Figure 5]
• LCx supplies inferolateral aspect of left
ventricle (LV)
• Obtuse marginals (OM) branches of LCx
extend anteriorly along inferolateral wall
of LV

Figure 7 A,
B, C & D

Top Left
Image: Conus
branch (yellow
arrow).
Figure 5 A & B Top Right
Image:
Left circumflex (LCx, arrows) anatomy. Obtuse Atrioventricular
marginals (OM) branches (arrows) of LCx extend nodal branch
anteriorly along inferolateral wall of left ventricle (LV). (yellow arrow).
Left anterior descending (LAD). Bottom Right
Images:
Sinoatrial nodal
branch (yellow
Right Coronary Artery Anatomy [Figure 6] arrows).
• Arises from right coronary cusp
• Courses in right atrioventricular groove
• Distal to angle of heart, divides into
posterior descending artery (PDA) and
posterior left ventricular branch (PLV) in
most people Coronary Artery Anomalies
• Origin
 Multiple ostia
 High takeoff
 Single coronary artery*
 Anomalous origin of coronary artery
from pulmonary artery (ALCAPA)*
 Origin of coronary artery or branch
from opposite or noncoronary sinus
and an anomalous
™™ Retroaortic
Figure 6 A & B
™™ Interarterial*
Right coronary artery (RCA). IVC = inferior vena cava. ™™ Prepulmonic
PDA = posterior descending artery. LV = left ventricle. ™™ Septal*
RV = right ventricle. PLV = posterior left venticle.
• Course
 Myocardial bridging*
 Duplication of arteries
• Termination
 Coronary artery fistula*
 Coronary arcade
Smaller Vessels [Figure 7]
 Extracardiac termination
• Conus branch
 Usually first branch off of RCA *May lead to compromise of blood flow
 Can have separate origin
 Supplies right ventricular outflow tract
(RVOT)
• Atrioventricular nodal branch
 Usually first branch off PLV
 Supplies AV node
• Sinoatrial nodal branch
 Can arise from RCA (59%), LCx
(39%), or both (2%)
 Supplies sinoatrial node

Coronary Artery Anatomy and Pathology Cardiac Imaging


1504
Benign Abnormalities in Coronary Benign Anomalies [Figure 10]
Artery Origin [Figure 8] • Prepulmonic LAD
• Atresia of proximal left main coronary
artery or LAD
• LAD arises from right coronary cusp
and courses anterior to right ventricular
outflow tract (RVOT)
• Likely represents hypertrophied conus
branch

Figure 10

Prepulmonic
left anterior
descending
(LAD) (red
arrow).
Figure 8 A, B & C RVOT = right
ventricular
Top Left Image: outflow tract.
High origin (yellow
arrow).
Top and Bottom
Right Images:
Separate ostia (red
arrows).
Potentially Malignant Anomalies
[Figure 11]
• Interarterial course
• Septal course
• Single coronary artery
• ALCAPA

Benign Anomalies [Figure 9] Figure 11


• Retroaortic coronary artery
 LCx or left main coronary artery arises Interarterial
from the right coronary cusp course of
RCA (arrow).
 RCA arises from left coronary cusp
• Vessel courses posterior to aorta between
aorta and left atrium

Potentially Malignant [Figure 12]


• Interarterial course of RCA
• Risk of sudden cardiac death
 Less than interarterial LM/LAD
• Causes
Figure 9 A & B  Acute takeoff angle
 Slit-like orifice
Benign anomalies. The left circumflex (LCx) courses
posterior to aorta between aorta and left atrium (red  Compression of intramural segment
arrows).  Extrinsic compression
• Debate about treatment
 Unroofing procedure
 Widening of ostia
 Reimplantation
 Do nothing

Cardiac Imaging Coronary Artery Anatomy and Pathology


1505
Figure 12

Interarterial
course of RCA
(arrows).
RCC = right Figure 14
coronary
cusp; LCC = Potentially
left coronary malignant.
cusp; NCC =
noncoronary
cusp.

Potentially Malignant [Figure 15]


• Anomalous origin of left main coronary
artery from PA (ALCAPA)
• Bland-Garland White syndrome
Potentially Malignant [Figure 13] • Most present in infancy
• Anomalous left main coronary artery • Severity depends on collateralization and
• Less common than anomalous RCA direction of blood flow
• Has interarterial course in 75%
• High incidence of sudden cardiac death
• Needs surgical treatment

Figure 15

Anomalous
origin of left
Figure 13 main coronary
artery from
Anomalous pulmonary
left main artery
(LM) (ALCAPA)
coronary (arrow). LM =
artery left main.
(arrow).

Abnormalities in Course Myocardial


Bridging [Figure 16]
Potentially Malignant [Figure 14] • Common finding
• Single coronary artery  >50% on autopsy
 Extremely rare  Most superficial
 Can be associated with congenital • Can lead to ischemia and infarct in some
heart disease patients
 Increased risk for sudden death  Delayed coronary artery relaxation
™™ Major coronary branch with during early diastole
interarterial course • Bridged segment free of atherosclerotic
™™ Proximal stenosis disease
• Septal course
 Intramyocardial
 Less dangerous than interarterial
course

Figure 16 A & B

Abnormalities in course myocardial bridging. Courtesy


of Jamie Brown, MD.

Coronary Artery Anatomy and Pathology Cardiac Imaging


1506
Abnormalities in Termination: Coronary Kawasaki Disease [Figure 19]
Artery Fistula [Figure 17] • Mucocutaneous lymph node syndrome
• Congenital or acquired  Acute self-limited vasculitis
• RCA (60%) > LCA (40%)  Fever, nonexudative conjunctivitis,
• Artery dilated and tortuous erythema of lips and oral mucosa, skin
• Drainage site important rash, and cervical lymphadenopathy
• Right drainage • Cardiac manifestations
 Most common  Myocarditis (50%)
 Extracardiac left to right shunt  Pericarditis
• Left drainage  Coronary artery aneurysms (3%–
 Rare 15%)
 Mimics aortic insufficiency  Arrhythmias
• May lead to myocardial ischemia  Ischemia
• Long-term treatment directed at cardiac
issues
 Coronary artery bypass graft (CABG)
 Transplant

Figure 17

Drainage site
important (red
arrow). LAD
= left anterior
descending;
RV = right
ventricular.
Figure 19 A & B
Kawasaki disease. Coronary artery aneurysms
Acquired Coronary Artery (arrows).
Abnormalities: Coronary Artery
Aneurysms [Figure 18]
• Coronary diameter >1.5X normal Coronary Artery Pseudoaneurysm
[Figure 20]
• Causes • Contained rupture of coronary artery
 Atherosclerosis • Less common
 Kawasaki disease • Most often seen in venous or arterial graft
 Congenital at site of anastomosis
 Connective tissue disease • Can occur from trauma or infection
• Saphenous vein grafts
 Accelerated atherosclerosis
 Distant from anastomosis

Figure 20

Figure 18 RCA pseudo-


aneurysm.
Acquired Contained
coronary rupture of
artery coronary artery
abnormalities: (red arrow).
coronary
artery
aneurysms.

Cardiac Imaging Coronary Artery Anatomy and Pathology


1507
Summary
• Coronary artery anatomy is widely
variable
• No real normal exists
• Anomalies can be considered congenital
or acquired
• Congenital anomalies can be
differentiated in different ways
 Anomalous course
 Anomalous origin
 Anomalous termination
 Benign
 Potentially malignant

Acknowledgments
• A special thanks to Allen Burke, MD who
provided the majority of the pathologic
images provided in this lecture.

Coronary Artery Anatomy and Pathology Cardiac Imaging


1508
Coronary Artery Disease

Seth J. Kligerman, MD
Coronary Arteries Coronary Artery Disease
• Epicardial vessels • Leading cause of death in men and
• Provide oxygenated blood to myocardium women in the Western World
• Divisions • Around 450,000 people die each year in
 Left main US from coronary artery disease (CAD)
™™ Left anterior descending (LAD) • 1.1 million myocardial infarcts/year in US
™™ Left circumflex (LCx)  40% fatal
 Right • Heart disease costs $316.4 billion/year

CTA Acquisition Corinary Artery Disease– An


• ECG gated for cardiac cycle Inflammatory Process
™™ Retrospective • Arterial endothelium exposed to
™™ Prospective  Bacterial products
™™ High pitch helical  Lipids
 Proinflammatory cytokines
Retrospective Gating  Vasoconstrictor hormones
• Helical acquisition • Activation of endothelium
• Patient scanned during entire cardiac  Overexpression of adhesion molecules
cycle • Blood leukocytes (mononuclear
 Dose higher phagocytes and T-cells)
™™ Dose modulation  Adhere to endothelium
™™ 6–25 mSv  Migrate into intima
 Functional data • Leukocytes release inflammatory
mediators and cytokines
Prospective Gating [Figure 1]
 Additional leukocytes and
• Axial acquisition macrophages recruited
 Step and shoot  Smooth muscles cells (SMC)
• Beam turned off during systole proliferate
 Lower dose
™™ 3–6 mSv Coronary Artery Histology
 No functional information • 3 layers
Figure 1 ™™ Intima
Curved ™™ Media
multiplanar ™™ Adventitia
reformat of the
right coronary Intima
artery shows a • Endothelial cells (E) lining the lumen
stair-stepping
• Bound by internal elastic lamina (IEL)
artifact (white
arrows) that can  Fenestrated sheet of elastin fibers
be seen with • Thin subendothelial (SE) space in between
prospective “step  Elastin and collagen fibers
and shoot” gating.  Smooth muscle cells (SMCs)
Newer scanners
with large
detector arrays Role of Endothelium
and improved • Nonthrombogenic surface
postprocessing • Vascular tone and remodeling
software have led • Platelet activation
to a significant • Thrombus
decrease in this
artifact.  Creation
 Lysis
• Lipid metabolism
High Pitch Helical Acquisition • Monocyte adhesion
• Newer method of acquisition
• Helically acquisition during 1 short
segment of single heart beat
 Dual source
 Pitch >3
• Doses <1 mSv

Cardiac Imaging Coronary Artery Disease


1509
Coronary Artery Histology Fibrous Cap
• Media • Overlies core of foam cells, lipid, and NC
 Predominantly SMC • SMCs
• Adventitia  Deposition of extracellular connective
 External elastic lamina tissue matrix
 Nerve fibers  Helps form FC
 Connective tissue • Inflammatory cells
 Vasa vasorum • Thickness varies
 Thick cap
Nonpathologic Findings  Thin cap
• Diffuse intimal thickening ™™ Inflammatory cells thin cap
 Normal aging process ™™ Prone to rupture
 Seen in fetuses and infants ™™ “Vulnerable” plaque
 Well-organized structure
 Pathologic factors absent Necrotic Core
• Intimal xanthoma “fatty streak” • Avascular
 Deposition of lipids • Hypocellular
 Macrophages infiltrate intima • Contains
™™ Form “foam cells”  Free cholesterol
 SMC proliferation  Cholesterol crystals
 Esterified cholesterol
Pathologic Intimal Thickening Grade III ™™ Circulating lipids
• Intermediate lesion ™™ Foam cell death
• Increasing • Size of NC directly related to likelihood of
 SMC proliferation plaque rupture
 Macrophage infiltration
 Extracellular lipid Fibroatheroma on CT [Figure 3]

Fibrous Cap Atheroma, aka


Fibroatheroma [Figure 2]
• Lipid core plaque
• Cellular proliferation and death continues
• Two main components
 Fibrous cap (FC)
 Necrotic core (NC)

A. Calcified B. Mixed

Figure 3 A, B & C

Multiplanar
CT reformats
from 3 different
patients with
coronary artery
atherosclerotic
disease showing:
A. completely
calcified
atherosclerotic
plaque (red
arrow); B. mixed
calcified and
noncalcified
atherosclerotic
Figure 2 plaque (yellow
arrow); and
Cross section of a fibroatheroma showing a thick C. completely
fibrous cap (FC) composed of smooth muscle cells, noncalcified plaque
extracellular matrix, and inflammatory cells overlying (white arrow).
C. Noncalcified
an avascular, hypocellular necrotic core (NC) composed
of free cholesterol, cholesterol crystals, and esterified
cholesterol.

Coronary Artery Disease Cardiac Imaging


1510
Coronary Artery Calcification
• Similar to bone formation
• Osteoblast-like cells derived from SMCs
• Endochondral ossification
• Lipid core mineralization
• Fibrous tissue microcalcification

Coronary Artery Calcification


• Strong predictor of incident coronary
artery disease Figure 5 A & B
• Risk stratification Long axis (left image) and transverse (right image)
• Wide-spread screening debate images through a coronary artery show extensive
• Only detects calcified and mixed plaque predominantly noncalcified plaque in the left
circumflex coronary artery (red arrows) leading to
Calcium Score [Figure 4]
expansion of the vessel wall without narrowing of the
lumen (L) consistent with positive remodeling.

Negative Remodeling [Figure 6]


• Luminal stenosis
• Many plaques with positive remodeling
progress to negative remodeling
• Stable angina

Figure 6
Figure 4 A & B
Image from a gated noncontrast CT of the coronary Long axis
arteries for calcium scoring demonstrate extensive image of the
calcification of the left anterior descending coronary left anterior
artery (arrows). Gated contrast CT coronary descending
angiography in the same patient shows the extensive coronary
amount of noncalcified plaque leading to areas of artery shows a
narrowing (arrows) associated with the areas of large focus of
calcification. predominantly
noncalcified
plaque (white
arrow) leading
to narrowing
Potential Fibrous Cap Atheroma and stenosis of the vessel lumen consistent with
negative remodeling.
Outcomes

Coronary Artery Stenosis – Progressive


Reduction in Diameter
• Minimal: <25%
• Mild: 26%–50%
• Moderate: 51%–70%
• Severe: 71%–99%
Plaque growth • Occlusion: 100%
• Outward growth
 Positive remodeling
• Inward growth
 Negative remodeling

Positive Remodeling [Figure 5]


• Outward compensatory remodeling
 Lumen initially uncompromised
• Can grow for decades
 Narrowing of at least 50%–70% to
cause flow limitation
• “Vulnerable plaques”
 Prone to rupture
 Acute coronary syndrome

Cardiac Imaging Coronary Artery Disease


1511
Minimal Narrowing <25% [Figure 7] Severe Stenosis: 71%–99% [Figure 10]

Figure 7 A, B & C

Long axis (A) and


transverse (B) images
of the left anterior
descending coronary Figure 10 A & B
artery show a trace Transverse image of the right coronary artery shows
amount of predominantly a large focus of noncalcified plaque (white arrow)
calcified plaque (arrows) leading to near complete occlusion of the vessel
leading to minimal lumen consistent with a severe stenosis as defined as
narrowing of the vessel, narrowing of the vessel diameter between 71%–99%.
defined as narrowing less
than 25% of the vessel
diameter.
Occlusion: 100% [Figure 11]

Mild Stenosis: 26%–50% [Figure 8]

Figure 11 A & B
Long axis maximum intensity projection (MIP)
image of the left anterior descending coronary
artery demonstrates a large focus of predominantly
Figure 8 A & B
noncalcified plaque (white arrow) leading to complete
Transverse image of the right coronary artery shows occlusion of the vessel lumen.
a large focus of noncalcified plaque (white arrow)
leading to mild narrowing of the vessel lumen as
defined as narrowing of the luminal diameter between
26%–50%.
Plaque growth
• Lipid-rich plaques
Moderate Stenosis: 51%–70%  Expansion of NC
[Figure 9]  Increasing calcification
 Intraplaque hemorrhage
 Healed rupture

Figure 9 A & B
Transverse image of the left anterior descending
coronary artery shows a large focus of noncalcified
plaque (white arrow) leading to moderate narrowing
of the vessel lumen as defined as narrowing of the
luminal diameter between 51%–70%.

Coronary Artery Disease Cardiac Imaging


1512
Neovascularity and Intraplaque
Hemorrhage [Figure 12]
• Plaques acquire microvascular network
• Immature vessels weak and leaky
• Blood components extravasate into plaque
 Cholesterol crystal production
 Oxidant activities
 Proteolytic activities

Figure 12

A 150-
micrometer
thick
section of a
fibroatheroma
stained with
Ulex shows
multiple
small vessels Figure 14
(white arrow) High-power cross section of a ruptured fibroatheroma
growing shows disruption of the fibrous cap (black arrow) with
toward the the highly thrombogenic necrotic core (red arrow)
necrotic core. These vessels are weak and can pouring into the vessel lumen.
hemorrhage within the plaque, leading to plaque
expansion.

Plaque Rupture Outcomes


Thin-cap Fibrous Atheroma [Figure 13] • Occlusive thrombus
• Inflammation thins fibrous cap  Acute infarction
 Diameter <65 μ  Sudden cardiac death
• Precursor of plaque rupture  Plaque organization
 >95% of plaque ruptures occur with ™™ Recanalized
diameters <65 μ ™™ Chronically thrombosed

Figure 13 Occlusive Thrombus: Acute Myocardial


Cross section Infarction [Figure 15]
of a large • Plaque rupture or erosion leading to
fibroatheroma complete occlusion of a coronary artery.
shows that the • Negative remodeling leading to complete
fibrous cap
(arrow) has
occlusion less common
thinned due to
infiltration with
inflammatory
cells and
mediators.
This leads
to increased
vulnerability
of the plaque
as the highly thrombogenic necrotic core is separated
from the lumen of the vessel only by a thin fibrous cap.

Figure 15 A & B

Left Image: Plaque rupture leading to complete


occlusion of left anterior descending coronary artery
Acute Plaque Rupture Type VI lesion (yellow arrow).
[Figure 14]
Right Image: Resultant hypoperfusion demonstrated
by subendocardial perfusion defect (white arrows) in
• Fibrous cap disrupts left anterior descending territory.
• Highly thrombogenic material pours into
lumen
• Most prevalent
 Proximal coronary arteries
 Near branch points

Cardiac Imaging Coronary Artery Disease


1513
Occlusive Thrombus Chronic Total Lipid Poor Plaques
Occlusion [Figure 16] • Poorly understood
• More common in women
• Progressive intimal thickening
Figure 16 • Repeated erosions

Chronic Plaque Erosion


occlusion of • Erosion of endothelial layer
left anterior
descending • Fibrous cap intact
coronary  Luminal thrombi not due to plaque
artery rupture
(yellow • Mechanism unknown
arrow) with
associated Conclusion
findings of
a chronic • Coronary artery disease is complex
myocardial infarction with calcified apical aneurysm inflammatory process
(white arrows). • Luminal narrowing or occlusion occur by
variety of mechanisms
• Radiologic imaging of CAD still in infancy
• Key is distinction between plaques
Nonocclusive Thrombus [Figure 17]  RF intravascular ultrasound
• Progressive luminal narrowing  Not yet possible on CT, MR, or PET
• Stable or unstable angina
Acknowledgments
• A special thanks to Allen Burke, MD who
provided the majority of the pathologic
images provided in this lecture.
Figure 17

Axial
multiplanar
reformat of
the proximal
left anterior
descending
coronary
artery in a
35-year-old
man shows a
large focus of
noncalcified
plaque (white
arrow) leading to severe narrowing of the vessel
(black arrow).

References
1. Achenbach S, Marwan M, Ropers D, et al. Coronary computed tomography angiography with a consistent
dose below 1mSv using prospectively electrocardiogram-triggered high-pitch spiral acquisition. European
Heart Journal 2010;31:340–346.
2. Libby P, DiCarli M, Weissleder R, et al. The vascular biology atherosclerosis and imaging targets. J Nucl
Med. 2010;51:1S–5S.
3. Dalager S, Paaske WP, Kristensen IB, et al. Artery related differences in atherosclerosis expression.
Stroke. 2007;38:2698–2705.
4. Detrano R, Guerci AD, Carr JJ, et al. Coronary calcium as a predictor of coronary events in four racial or
ethnic groups. N Engl J Med. 2008;358:1336–1345.
5. Virmani R, Burke AP, Farb A, Kolodgie FD. Pathology of the vulnerable plaque. J Am Coll Cardiol.
2006;47:13–18.
6. Burke AP, Kolodgie FD, Farb A, et al. Healed plaque ruptures and sudden coronary death evidence that
subclinical rupture has a role in plaque progression. Circulation 2001;103:934–940.
7. Michel JB, Virmani R, Arbustini E, et al. Intraplaque haemorrhages as the trigger of plaque vulnerability.
Eur. Heart Journal 2011; 32: 1977–1985.
8. Farb A, Burke AP, Tang AL, et al. Coronary plaque erosion without rupture into a lipid core. A frequent
cause of coronary thrombosis in sudden coronary death. Circulation. 1996;93:1354–63.

1514
Valvular Heart Disease

Jean Jeudy, MD
Objectives Valve Anatomy – Fibrous skeleton
• Differentiate between the 2 types of heart • Valves are reinforced by the cardiac
valves with regard to their anatomy and skeleton made up of dense connective
physiology tissue
• Describe specific causes of individual  Attachment site for the valve leaflets
valve disorders as well as multi-valvular  Electrical insulation between atria and
disease ventricles
• Become familiar with quantitative  4 annuli fibrosi
imaging techniques to characterize valve  Membranous septum
abnormalities
Valve Anatomy – Atrioventricular Valves
Valvular Heart Disease [Figure 1]
• Accounts for 10%–20% of all cardiac • Mitral (bicuspid)
surgical procedures in the US  Annulus
• A common spectrum of pathology is often  2 leaflets
observed  2 papillary muscles
• Valvular disease can come to clinical  2 sets of chordae tendinae
attention due to stenosis, insufficiency  Commissures
(regurgitation or incompetence), or both ™™ Anterolateral
 Stenosis is the failure of a valve to ™™ Posteromedial
open completely, which impedes ™™ Subdivision based on 3 scallops of
forward flow the leaflet
 Insufficiency results from failure of ™™ A1, A2, A3 and P1, P2, P3
a valve to close completely, thereby
allowing reversed flow

Valvular Stenosis Figure 1


• Acquired stenoses of the aortic and mitral
Mitral valve
valves account for approximately two- is a bicuspid
thirds of all cases of valve disease valve that is
• Valvular stenosis is almost always due to composed of
a chronic abnormality of the valve cusp a semicircular
that becomes clinically evident after many anterior
leaflet and
years a crescentic
• Relatively few disorders produce valvular posterior
stenosis leaflet and
is anchored
Valvular Insufficiency on the
mitral valve
• In contrast, valvular insufficiency can annulus.
result from intrinsic disease of the valve Both leaflets are connected to the left ventricular
cusps or damage to or distortion of the papillary muscles by the chordae tendinae.
supporting structures (e.g., the aorta,
mitral annulus, tendinous cords, papillary
muscles, ventricular free wall)
• Valvular insufficiency has many causes
and may appear acutely, as with rupture
of the cords, or chronically in disorders Valve Anatomy – Atrioventricular Valves
associated with leaflet scarring and • Tricuspid
retraction  3 leaflets
™™ Anterior, posterior, and septal
• Commissure
 Named by the adjoining leaflets
• 3 papillary muscles
• Chordae tendinae

Cardiac Imaging Valvular Heart Disease


1515
Valve Anatomy – Semilunar Valves Aortic Stenosis – Severity of Disease
• Aortic and pulmonary valves • It is important to determine the
• 3 semilunar cusps severity of aortic stenosis based upon
 Left, right hemodynamic measurements
 Anterior/posterior • The outcome of these patients is related
• Sinuses of Valsalva to the severity of the lesion at diagnosis
 Ostia from aorta emptying into
coronaries

Left-sided Valvular Disease – Aortic


Valve

Aortic Stenosis
• With the exclusion of systemic
hypertension, aortic stenosis is the
second most common potentially fatal Aortic Valve Area
or fatal heart disease after coronary • Measured by planimetry
atherosclerosis  Aortic valve area =
 147.2 mm2 or 1.47 cm2
Aortic Stenosis – Bicuspid Aortic Valve
• Most common congenital cardiac Aortic Insufficiency
abnormality, occurring in approximately • Most often due to direct valve
2% of the population abnormalities
• Younger patients, rarely stenotic at birth  Rheumatic
• Develop fibrosis and calcification with  Endocarditis
increasing age with resulting stenosis  Bicuspid valve
 By age 50 • Dilatation of the aortic root
 Marfan
Aortic Stenosis – Degenerative fibrosis
• Senile type >60 Aortic Insufficiency
• Tri-leaflet valves • Usually due to severe calcific aortic
• Progressive degenerative changes with stenosis (AS) where valve is fixed in
calcific deposits at the base of the aortic location
valve cusps • Rheumatic fever – cusps become
 May be associated with fusion of infiltrated with fibrous tissue and cause
commissures making it difficult to retraction
distinguish from bicuspid • Infectious endocarditis inflammatory
• Frequently found in conjunction with disorders of the aortic root
coronary artery and cerebrovascular
disease Progressive Cycle of Disease
• When the aortic annulus becomes greatly
Aortic Stenosis – Rheumatic Heart dilated, the aortic leaflets separate and
Disease aortic regurgitation (AR) may ensue
• Results from adhesions and fusions of the • Dissection of the diseased aortic wall may
commissures and cusps occur and aggravate the AR
• Calcific nodules develop on valve surfaces • Dilation of the aortic root may also have
• Thickening of valve leaflets and tensor secondary effects on the aortic valve
apparatus because dilation causes tension and
• More commonly involves the mitral valve bowing of the individual cusps, which may
(65%–70%) along with the aortic valve in thicken, retract, and become too short to
another 25% of cases close the aortic orifice

 outflow obstruction leads to  Left


Ventricular Hypertrophy
• Hypertrophied hearts are more sensitive
to diffuse subendocardial ischemic injury,
which may result in both systolic and
diastolic dysfunction
• Obstruction progresses to a critical level,
the high afterload “overwhelms” the left
ventricle and systolic function begins to
decrease

Valvular Heart Disease Cardiac Imaging


1516
Aortic Insufficiency [Figure 2]
• Intensification of AR, leads to further
dilatation of the ascending aorta and thus
leading to a vicious circle
 Regurgitation leads to regurgitation

Figure 2

Aortic
regurgitation,
regardless
of its cause,
produces
dilation and
hypertrophy
of the left
ventricle,
Figure 3 A to E
dilation of the
mitral valve Endocarditis
ring, and of native but
sometimes previously
hypertrophy damaged or
and dilation of otherwise
the left atrium. abnormal
valves, is
caused most
commonly
Infective Endocarditis (50%–60% of
• Infection of the heart valves with cases) by
Streptococcus viridans, which is part of the normal
development of vegetations flora of the oral cavity. In contrast, more virulent
 Thrombotic debris and organisms S. aureus organisms commonly found on the skin
• Often associated with valve destruction can infect either healthy or deformed valves and are
responsible for 10%–20% of cases overall; S. aureus
Infective Endocarditis [Figure 3] is the major offender in intravenous drug abusers with
• Acute infective endocarditis. The roster of the remaining
bacteria includes enterococci and the so-called HACEK
 Infection of normal heart valves by group (Haemophilus, Actinobacillus, Cardiobacterium,
highly virulent organisms Eikenella, and Kingella), all commensals in the oral
• Subacute cavity.
 Insidious infection of deformed valves
by less virulent organisms

Noninfectious Endocarditis Rheumatic Heart Disease – Worldwide


• Marantic endocarditis Deaths [Figure 4]
• Occur in the setting of connective tissue
diseases
 Systemic lupus erythematosus
(Libman-Sacks)
 Antiphospholipid antibody syndrome

Left-sided Valvular Disease – Mitral


Valve

Mitral Stenosis
• Rheumatic heart disease is by far most
common and has a predilection for left-
sided valves with mitral being most
common Figure 4 A & B
• Estimated 15 million people affected
Although rates have declined significantly in North
worldwide America and many other parts of the world, rheumatic
heart disease remains an important public health
problem, affecting an estimated 15 million people
worldwide.

Cardiac Imaging Valvular Heart Disease


1517
Rheumatic Fever Pathophysiology
• Rheumatic fever is an acute, • Left atrium progressively dilates and may
immunologically mediated, multisystem develop mural thrombi in the appendage
inflammatory disease that occurs a or along the wall
few weeks after an episode of group A • Long-standing congestive changes in the
streptococcal pharyngitis  lungs may induce pulmonary vascular and
• Acute rheumatic carditis is a frequent parenchymal changes and in time lead to
manifestation during the active phase of right ventricular hypertrophy
rheumatic fever and may progress over
time to chronic rheumatic heart disease Pathophysiology [Figure 6]
(RHD), of which valvular abnormalities • Microscopically, there is organization of
are key manifestations the acute inflammation
• Subsequent diffuse fibrosis and
Progression to Rheumatic Heart Disease neovascularization that obliterate the
• Generally a slowly progressive disease, originally layered and avascular leaflet
with a latency period as long as 20–40 architecture
years between the initial infection and the
onset of clinical symptoms
• Inflammation of the endocardium and
the left-sided valves typically results in
fibrinoid necrosis within the cusps or
along the tendinous cords

Mitral Stenosis – Rheumatic Heart


Disease
• Small (1- to 2-mm) vegetations, called
verrucae, along the lines of closure
• Subsequent scarring leads to valve
deformity and obliteration of the normal
leaflet architecture
 Fibrosis
 Neovascularization Figure 6 A, B, C & D
 Increased collagen deposition
Spongiosa element such that the leaflet itself consists
entirely or nearly entirely of the fibrosa element.
Mitral Stenosis – Rheumatic Heart
Disease [Figure 5]
• Primary anatomic changes of the mitral Pathophysiology
valve in chronic RHD are: • Primary hemodynamic consequence
 Leaflet thickening of mitrial stenosis (MS) is a pressure
 Commissural fusion and shortening gradient between the left atrium and left
 Thickening and fusion of the tendinous ventricle in diastole
cords • Elevated left atrial pressure is reflected
backward, causing an increase in
pulmonary venous, capillary, and arterial
pressures and resistance
Figure 5 Mitral Valve Orifice
Fibrous • Severity of mitral valve obstruction is
bridging related to degree of valve opening in
across the diastole
valvular  Normal adults, mitral valve orifice
commissures
and (MVO) is 4–6 cm2
calcification  Mild MS – approximately 2 cm2
create “fish  Severe MS – MVO reduced to 1 cm2
mouth” or
“buttonhole” Mitral Insufficiency
stenoses.
End-stage • Pure mitrial regurgitation (MR)
leaflets (no stenosis) is the most common
may be so dysfunctional cardiac valve disorder, and,
adherent and in contrast to MS, it has many different
rigid that they
causes
cannot open
or shut. • MR can occur secondary to any
abnormalities of the mitral valve
apparatus
Valvular Heart Disease Cardiac Imaging
1518
Mitral Insufficiency – Morphologic Mitral Insufficiency – Myxomatous
Abnormalities Disease [Figure 7]
• Valve leaflets • Key histologic change in the tissue is
• Mitral valve annulus called myxomatous degeneration
• Tensor apparatus

Abnormalities of the Valve Leaflets


• Flail leaflet due to myxomatous disease
(mitral valve prolapse)
• Secondary prolapse of mitral leaflets
without primary myxomatous
degeneration
 Dilated left ventricle (LV)
 Elongation/rupture of papillary
muscles (infection, ischemia, blunt
injury) Figure 7 A & B
 Rheumatic heart disease
Normal heart valve (D) and myxomatous mitral valve
Abnormalities of the Mitral Valve (E) (Movat pentachrome stain, in which collagen is
yellow, elastin is black, and proteoglycans are blue).
Annulus In myxomatous valves, collagen in the fibrosa is
• Dilatation loose and disorganized, proteoglycans (asterisk) are
 Cardiomyopathy deposited in the spongiosa, and elastin in the atrialis is
• Calcification disorganized.
Courtesy of Robbins & Cotran.
 Mitral annular calcification
 Marfan and Hurler syndromes
™™ Defect of fibroskeleton of the heart
 Chronic renal failure Mitral Insufficiency – Myxomatous
Disease
Abnormalities of the Tensor Apparatus
• Chordae tendinae rupture – to chronic LV
dilatation, trauma, spontaneous rupture,
infective endocarditis, or acute rheumatic
fever
• Papillary muscle rupture – due to acute
myocardial infarction or severe ischemia
or trauma
 In transient ischemia, can result in
pap mm dysfunction and transient

Mitral Insufficiency – Myxomatous


Disease
• Systolic ballooning of the mitral leaflets
• Affected leaflets often enlarged,
redundant, thick, and rubbery Mitral Annular Calcification
• Associated chordae may be elongated, • Mitral annular calcification is a common
thinned, or even ruptured disorder that rarely leads to mitral
• Annulus may be dilated stenosis
• Tricuspid, aortic, or pulmonary valves may
• Often seen with end-stage renal disease
also be affected
Mitral Insufficiency – Clinical
Presentation
• Acute
 Rupture of tensor apparatus
 Leaflet perforation (endocarditis)
 Acute shortness of breath (SOB), florid
pulmonary edema
• Chronic
 Left atrial (LA) enlargement
 Atrial fibrillation
 Dyspnea on exertion (DOE)

Cardiac Imaging Valvular Heart Disease


1519
Acute Mitral Insufficiency Causes of Tricuspid Abnormality
• Hemodynamic changes in acute MR are • Congenital atresia or stenosis of the valve  
more severe than those in chronic MR • Right atrial or metastatic tumors
due in part to the lack of time for the left • Carcinoid syndrome, which may cause
atrium and left ventricle to adapt to the stenosis, although tricuspid regurgitation
mitral regurgitation is more common
• Patients who develop acute severe • Bacterial endocarditis, particularly in
MR usually have a normal-sized left association with a permanent pacemaker
atrium, with normal or reduced left atrial lead or a prosthetic valve
compliance
Tricuspid Stenosis
Acute Mitral Insufficiency • Mean pressure gradient between the right
• Left atrial pressure rises abruptly, leading atrium (RA) and right ventricle (RV) as
to low as 2 mmHg, is sufficient to establish
 Pulmonary edema the diagnosis of tricuspid stenosis
 Marked elevation of pulmonary • Since the pressures in the right-sided
vascular resistance cardiac chambers are usually low, even
 Right-sided heart failure a gradient as small as 5 mmHg can be
sufficient to elevate mean right atrial
Chronic Mitral Insufficiency [Figure 8] pressure
• Functional adaptations have time
to develop and typically preserve Tricuspid Insufficiency [Figure 9]
hemodynamic stability • Tricuspid regurgitation (TR) is commonly
• Atrial fibrillation is common in patients NOT due to intrinsic involvement of the
with chronic MR valve itself
 Associated with age and left atrial  Primary TR
dilation • Dilation of the right ventricle and of the
 Its onset is a marker for disease tricuspid annulus
progression  Secondary (functional) TR
• Severe MR develops in only a small • By definition, when tricuspid regurgitation
percentage of these patients is functional, the tricuspid valve leaflets
 Infective endocarditis are morphologically normal
 Rupture of the chordae tendinea
Figure 9

Almost all
causes of
Figure 8 acquired
disease are
Patients rare except
with atrial for occasional
fibrillation cases due
have an to trauma,
adverse infective
outcome endocarditis
compared in intravenous
with patients drug users,
who remain and valve
in sinus injury due
rhythm. to the jet
through
a small
ventricular
septal defect.
Right-sided Valvular Disease – Tricuspid
Valve

Tricuspid Stenosis and Insufficiency Tricuspid Insufficiency


• Tricuspid stenosis is an uncommon • Since the right atrium is relatively
valvular abnormality that still is most compliant, there are often no major
commonly the result of rheumatic heart hemodynamic consequences with mild or
disease moderately severe TR
• If tricuspid valve is made stenotic on the • When TR is severe, right atrial and venous
basis of rheumatic heart disease, the pressure rise and can result in the signs
mitral valve is also always stenotic, never and symptoms of right-sided heart failure
purely regurgitant

Valvular Heart Disease Cardiac Imaging


1520
Tricuspid Disease – Rheumatic Heart Pulmonary Stenosis – Rheumatic Heart
Disease Disease
• Rheumatic fever may rarely involve the • Rheumatic inflammation of the pulmonic
tricuspid valve directly valve is very uncommon
 Scarring of the valve leaflets and/or • Usually associated with involvement of
chordae tendinea other valves
 Limited leaflet mobility and either • Rarely leads to serious deformity
isolated TR or a combination of TR and
tricuspid stenosis (TS) Pulmonary Stenosis – Classification
• Rheumatic involvement of the mitral, and • Mild when the peak gradient across the
often aortic, valves coexist valve <30 mm Hg  
• Moderate stenosis is defined as a peak
Carcinoid Heart Disease [Figure 10] gradient of 30–50 mm Hg
• White, fibrous carcinoid plaques • Severe pulmonic stenosis is defined by a
• Usually deposited on the ventricular peak jet velocity of greater than 50 mm
surfaces of the tricuspid valve and cause Hg
the cusps to adhere to the underlying RV
wall, producing TR Pulmonic Insufficiency
• In the adult, the most common cause
for moderate pulmonary insufficiency is
pulmonary hypertension that is primary or
secondary to left heart failure

Pulmonic Insufficiency – Carcinoid


Heart Disease
• Rarely produces mixed pulmonary
Figure 10 A & B insufficiency and stenosis that is always
Left Image: Characteristic endocardial fibrotic lesion
combined with tricuspid involvement
involving the right ventricle and tricuspid valve. • Predominantly regurgitant
Right Image: Microscopic appearance of carcinoid • Can also cause stenosis in the setting of
heart disease with intimal thickening. Movat stain valve thickening
shows myocardial elastic tissue (black) underlying
the acid mucopolysaccharide-rich lesion (blue-green). Infectious Endocarditis – Pulmonic
Tricuspid regurgitation or the combination of tricuspid
regurgitation and tricuspid stenosis is an important • Pulmonary valve endocarditis associated
feature of the carcinoid syndrome. Courtesy of with valvular vegetations is the rarest
Robbins. form of endocarditis
• Often diagnosed only by transesophageal
echocardiography (TEE)

Drug-induced Valvular Disease Pulmonic Insufficiency – Other Causes


• Repair of tetralogy of Fallot  
Methysergide – association with • Other procedures aimed at reconstructing
tricuspid regurgitation or repairing the outflow tract may result
in damage to the valve
Pergolide (dopamine agonist)
may induce tricuspid regurgita- Summary
tion by a mechanism similar to • Rheumatic heart disease
that of carcinoid syndrome  Multi-valvular involvement – mitral >
Fenfluramine-phentermine – aortic > tricuspid >> pulmonic
combined use of the anorectic  More often stenotic, but can also have
drugs insufficiency
• Infectious endocarditis
Right-sided Valvular Disease – Pulmonic  Multi-valvular involvement – mitral >
Valve aortic > tricuspid >> pulmonic
 Primarily insufficient
Pulmonary Stenosis
• Almost all cases of valvular pulmonic
stenosis are congenital in origin, and most
cases occur as an isolated lesion
• Valve dysplasia
• Unicuspid/bicuspid pulmonary valve

Cardiac Imaging Valvular Heart Disease


1521
Left-sided Valvular Disease – Aortic
Valve
• Stenosis
 Bicuspid vs degenerative fibrosis
™™ Aortic valve area
 Rheumatic heart disease
• Insufficiency
 Rheumatic heart disease
 Endocarditis
 Complications of aortic disease

Left-sided Valvular Disease – Mitral


Valve
• Stenosis
 Rheumatic heart disease
™™ Most common
™™ Always involves mitral valve
 Mitral annular calcification
• Insufficiency
 Rheumatic heart disease
™™ Most common
™™ Always involves mitral valve
 Mitral annular calcification
 Mitral valve prolapse
 Myxomatous degeneration

Right-sided Valvular Disease – Tricuspid


Valve
• Stenosis
 Rheumatic heart disease
 Carcinoid syndrome
 Drug-induced valvular disease
 Mitral annular calcification
• Insufficiency
 Most commonly secondary –
“functional”
 Any significant cardiac enlargement
 Rheumatic heart ddisease
 Carcinoid syndrome
 Drug-induced
 Endocarditis

Right Sided Valvular Disease – Pulmonic


Valve
• Stenosis
 Usually congenital
 Rheumatic heart disease is
UNCOMMON
• Insufficiency
 Usually congenital
 Rheumatic heart disease is
UNCOMMON
 Carcinoid syndrome
 Drug-induced valvular disease
 Endocarditis

Valvular Heart Disease Cardiac Imaging


1522
Primary Cardiac Masses

Jean Jeudy, MD
Objectives Primary Cardiac Tumors – Malignant
• To outline the spectrum of benign and Tumors
malignant cardiac masses • Malignant neoplasms are classified by
• To illustrate imaging features (including tissue type
lesion location, internal density, surface  Mesenchymal (sarcoma)
outline and mobility) which suggest  Lymphoid (lymphoma)
specific underlying histopathology  Mesothelial (mesothelioma)
• To emphasize that metastatic (secondary) • Sarcoma (10%–25% of primary cardiac
and benign neoplasms are far more masses) – further unclassified, or:
common than primary neoplasms of the  Angiosarcoma
heart  Osteosarcoma
 Leiomyosarcoma
Primary Cardiac Tumors  Fibrosarcoma
• Rare  Myxosarcoma
 0.001%–0.03% of autopsy series  Rhabdomyosarcoma
• 100–1,000 times LESS prevalent than • Lymphoma (RARE as a primary)
secondary cardiac neoplasms
Secondary Cardiac Tumors
Primary Cardiac Tumors – Clinical • Metastatic disease to the heart
Presentation  12% of autopsies with widespread
• Clinical presentation in children malignancy
 Hypoxia (50%)  20–40x more common than primary
• Clinical presentation in adults cardiac tumors
 Dyspnea (1/3)  Lung cancer
 Embolism (usually central nervous  Lymphoma
system (CNS) • Tumorlike lesions
 Dysrhythmias  Thrombus
 Congestive heart failure (CHF);  Valvular vegetations
myocardial infarction (MI)
 Tamponade Role of Imaging
• Location, size, shape/surface, mobility
Primary Cardiac Tumors • Extent
• Benign neoplasms are usually classified  Myocardium, valve, pericardium, lung
pathologically according to histologic parenchyma
features and cellular differentiation • Tissue characterization
 Muscle (rhabdomyoma) • Differentiation
 Fibrous (fibroma)  Benign vs malignant
 Vascular (hemangioma)  Nonsurgical vs surgical
 Fat (lipoma)
 Nervous (pheochromocytoma) Benign Cardiac Tumors – Myxoma
 Ectopic (teratoma)
Myxoma
Primary Cardiac Tumors – Benign • Female: male approximately 2:1
Tumors • Peak age 50 years
• Myxoma • Complications
 (50%–80% of primary cardiac masses  Stroke, transient ischemic (TIA)
in adults)  MI
• Rhabdomyoma (pediatric patients)  Peripheral emboli
• Lipoma ™™ Left atrium (LA)
• Fibroma  Pulmonary emboli
• Papillary Fibroelastoma ™™ Right atrium (RA)
• Hemangioma Rare
• Teratoma
• Pheochromocytoma

Cardiac Imaging Primary Cardiac Masses


1523
Myxoma [Figure 1]
• Most common benign cardiac tumor in
adults
• 75% found in the left atrium
• Majority attached to the fossa ovalis of
the interatrial septum
• Can also attach to the walls of the cardiac
chambers and valve surfaces
• 20% found in right atrium
• Rarely cases are observed in the
ventricles

Figure 2 A & B
These are 4-chamber, T1-weighted images. Point of
attachment is visible in up to 80% of cases.
On T1 images, myxomas, depending on their myxoid
content, may be either isointense to myocardium or
Figure 1 low attenuation.
Chest Figure 3
radiography
of both right Bright
atrial and blood cine,
left atrial 4-chamber
myxomas view, with
may be a large left
normal atria mass
although with blood
up to 50% turbulence
demonstrate as it flows
cardiomegaly around the
and are often interpreted as mitral stenosis. mass. Mass
With impending left atrial obstruction, evidence of does not
portal hypertension with septal lines and pleural touch valve
effusions. leaflets but
outflow to left
ventricle is
Myxoma – Computed Tomography affected.
• Well-defined, lobulated mass
• Low or heterogenous density Myxoma – Gross Histology
• Intracavitary • Pedunculated
• Interatrial septum attachment (+/-) • Firm, lobular surface
• Pleural effusions (+/-) • Myxoid, gelatinous contents
• No myocardial invasion
Myxoma – Magnetic Resonance • Friable – less common
[Figures 2 & 3]  Thrombogenic
• Point of attachment  Embolize (30%–40%)
• T1-weighted ™™ Central nervous system (CNS)
 Isointense to myocardium ™™ Coronaries
 Low signal: high myxoid content ™™ Aorta
• Tumor calcification ™™ Kidney
 Low signal ™™ Spleen
• Cine
 Arteriovenous (AV) mobility, Myxoma – Clinical Features
entrapment • Approximately 20% of patients with
cardiac myxoma are asymptomatic
• Classic clinical triad:
 Obstructive cardiac symptoms
 Embolic phenomena
 Constitutional symptoms

Myxoma – Sporadic
• Approximately 86% of these myxomas
occur in the left atrium
• 94% solitary
• Estimated risk of a second myxoma
developing after complete excision is
1%–3%

Primary Cardiac Masses Cardiac Imaging


1524
Myxoma – Nonsporadic
• 7% of cardiac myxomas
• Exhibit an atypical biologic behavior
 Multicentricity (45%)
 Atypical location (in cardiac chambers
other than the left atrium) (38%)
 Recurrence after surgical excision
(12%–22%)

Myxoma – Carney Complex Figure 4 A & B


• Autosomal dominant syndrome Lipomas are reported to be the second most common
 Cardiac, cutaneous, and mammary benign tumor of the heart in adults. They consist
myxomas of encapsulated mature adipose cells and fetal fat
 Spotty skin pigmentation cells. Their consistency is soft and they may grow
to a large size without causing symptoms, but may
 Endocrine overactivity (Cushing
cause dysrhythmias. Lipomatous hypertrophy of
syndrome, sexual precocity, the interatrial septum – non-neoplastic proliferation
acromegaly) of nonencapsulated fat cells – usually associated
 Psammomatous melanotic with advancing age and obesity can be a cause of
schwannoma supraventricular dysrhythmias, associated with older
age, and is defined as fat deposition >2 cm transverse
 Pigmented nodular adrenal disease diameter around (but sparing) fossa ovalis.
 Testicular neoplasms (especially
Sertoli cell)
• 2/3 of patients with Carney complex (CC)
develop cardiac myxoma Lipoma – Magnetic Resonance [Figure 5]
• Homogeneous high SI – T1
Myxoma – Main Points • Homogeneous low SI – T2
• Differential diagnosis
• Signal suppression on fat-saturated
 Atrial thrombus
• No evidence of enhancement with
 Papillary fibroelastoma
gadolinium
• Difficult to differentiate thrombus from
myxoma
 Myxomas – more likely to arise
anteriorly from the interatrial septum
+ myxoma enhance
 Thrombus – more likely located
posteriorly in the left atrium +
thrombus most often should not
enhance

Benign Cardiac Tumors – Lipoma Figure 5 A & B

Lipoma The MRI characteristics of cardiac lipoma are very


• Second most common benign cardiac specific (left image: axial T1, right image: fat
saturated). Lipomas have the same signal intensity as
tumor subcutaneous and epicardial fat; can be suppressed
 (Burke A, et al. AFIP 1996.) on fat-saturating pulse sequences. They demonstrate
• Mostly discovered incidentally no enhancement after contrast administration.
(asymptomatic); can be symptomatic of Usually, lipomas have sharp margins with the adjacent
in the pericardial space cardiac tissues.
 (Hananouchi GI, et al. 1990.)

Lipoma [Figure 4]
Lipoma – Main Points
• 10% of all cardiac tumors
• Differential diagnosis
 Less common than lipomatous
 Lipomatous infiltration
hypertrophy of interatrial septum
• Appearance is very characteristic on both
• Encapsulated proliferation of mature
CT and MR imaging
adipocytes
• Middle-aged, elderly
• May cause dysrhythmias
• Usually incidental
• Intracavitary, endocardial, myocardial or
epicardial/pericardial
• Usually right atrium or left ventricle
• Assess for caval obstruction

Cardiac Imaging Primary Cardiac Masses


1525
Benign Cardiac Tumors – Papillary Papillary Fibroelastoma – Main Points
Fibroelastoma • Differential diagnosis
 Vegetation
Papillary Fibroelastoma  Thrombus
• Most common primary tumor of the • Note:
cardiac valves  Vegetations – associated with valvular
 Cardiac valves (80%–90%) damage
™™ Aortic 35%  Papillary fibroelastoma (PF) usually
™™ Mitral 25% not associated with valvular damage
™™ Tricuspid 17% Klarich KW, et al. 1997.
™™ Pulmonary 13%
 Endothelial surfaces (10%) Benign Cardiac Tumors –Hemangioma
™™ LV apex
™™ Chordae tendinae Hemangioma
™™ Outflow tracts • Benign vascular tumors that can affect
patients of all age groups
Papillary Fibroelastoma • Approximately 5%–10% of benign tumors
• Benign endocardial papillomas • Most affected patients asymptomatic
• Rare (0.002%–0.3% incidence)  Discovered incidentally
• Small • Commonly present with dyspnea on
• Gelatinous branching fronds exertion but may also have chest pain,
 “Sea anemone” appearance right-sided heart failure, arrhythmias,
• Pedunculated (90%) pericarditis, or pericardial effusion
• Transthoracic echocardiography (TTE), • Can occur in any chamber; histologically
transesophageal echocardiography can be:
tee (TEE) are the preferred imaging  Capillary – composed of smaller
modalities capillary-like vessels
• Small and mobile  Cavernous – composed of multiple,
• Flutters, prolapses thin-walled, dilated vessels
 Arteriovenous – composed of thick-
Papillary Fibroelastoma – Magnetic walled dysplastic arteries, venous-like
Resonance [Figure 6] vessels, and capillaries
• Small (<1.5 cm), valvular, or endocardial
mass Hemangioma
• Too small to assess for contrast • Heterogeneously hypointense on T1-
enhancement weighted image
• Moves with heart motion • Hyperintense on T2-weighted image
• Differential diagnosis • Heterogeneous enhancement with
 Valvular vegetation gadolinium
 Thrombus • Intramural cardiac hemangiomas may
contain other tissue elements, especially
fat, and occasionally fibrous tissue, similar
to intramuscular soft-tissue hemangiomas

Benign Cardiac Tumors – Rhabdomyoma

Rhabdomyoma
• Cardiac rhabdomyomas represent up to
90% of cardiac tumors in infants and
children
 Usually discovered in patients less
than 1 year of age
• Benign myocardial hamartomas are
Figure 6 A & B strongly associated with tuberous
These lesions are small and therefore challenging to sclerosis
detect. Here are 2 images of a papillary fibroelastoma  Approximately 50% of patients with
attached to the right coronary cusp of the aortic
cardiac rhabdomyomas have tuberous
valve. Left image is a T1 oblique sagittal view of the
heart along the left ventricle outflow tract – lesion sclerosis
is isointense with myocardium. Quasi coronal bright
blood MR again filling defect at the base of right
coronary valve leaflet, iso to cardiac muscle. Right
image is a cut from a MR cine, also along the outflow
tract.

Primary Cardiac Masses Cardiac Imaging


1526
Rhabdomyoma Fibroma Imaging [Figures 8 & 9]
• Intramural, often multiple
• May regress spontaneously
• RV = LV
• 50% patients have tuberous sclerosis Figure 8
• Congenital hamartoma of myocytes
 “Spider cells” = vacuolated A 50-year-old
rhabdomyoma cells womand with
gradual onset
shortness
Rhabdomyoma [Figure 7] of breath.
• Usually detected <1 year old The chest
• Complications radiograph
 Fetal hydrops may appear to
be a common
 Tachyarrhythmias picture of
 Murmurs congestive
 CHF heart failure.
Symptoms may be abrupt or gradual in onset.

Figure 7 A & B
A 32-week gestation (male). Heart enlarged, almost
entirely taken up by a mass (largely anterior,
with compression of left atrium and left ventricle
posteriorly) – pulmonary hypoplasia, fetal hydrops. If Figure 9 A & B
nonlife-threatening, may be left alone because most of Four-chamber cardiac echo and four chamber.
these regress spontaneously.

Fibroma
Benign Cardiac Tumors – fibroma • Increased risk of cardiac fibromas in
fibroma patients with Gorlin (basal cell nevus)
• Second most common benign cardiac syndrome:
tumor in children  Multiple nevoid basal cell carcinomas
• 1/3 of patients are <1 year old of the skin, jaw cysts, and bifid ribs
 Probably congenital • Less than 14% of these patients have
• 15% in adolescents and adults cardiac fibromas
• 25%–50% of patients have tuberous
Benign Cardiac Tumors – Paraganglioma
sclerosis (TS)
• Arises within myocardium Paraganglioma
 Fibroblasts in collagen • Extremely rare neoplasms that arise from
• Arrhythmia, heart failure, SUDDEN DEATH intrinsic cardiac paraganglial (chromaffin)
• 30% asymptomatic cells, which are normally predominantly
• Intramural mass located within the atria
 Anterior wall RV or interventricular • Majority of reported cardiac
septum paragangliomas have been
• May obliterate cardiac chamber catecholamine-producing tumors
• Calcification common  Arterial hypertension
 Headache
 Palpitations
 Flushing

Cardiac Imaging Primary Cardiac Masses


1527
Paraganglioma [Figure 10] Malignant Cardiac Tumors –
• Up to 20% of patients with cardiac Angiosarcoma
paragangliomas have associated
paragangliomas in other locations Angiosarcoma [Figure 11]
• Approximately 5% of patients will have • Largest group of differentiated cardiac
osseous metastases sarcomas
• Histologic appearance identical to those of • Female = Male, 20–50 years old
extracardiac paragangliomas • Arise in myocardium, 90% RA
• Nests of paraganglial cells • Irregular vascular channels containing red
 “Zellballen” blood cells )RBCs)
• Grossly hemorrhagic

Figure 11

Angiosarcomas
represent the
largest group of
differentiated
Figure 10 cardiac sarcomas.
Unlike other
Surrounded by types of cardiac
sustentacular sarcomas, it has a
cells. strong right-sided
predominance.
Ninety percent
arise in the right
atrium, usually
with infiltration of
the pericardium
(can present
with syncope
and pericardial
tamponade).
Thirty percent
Benign Cardiac Tumors – Teratoma have lung mets at
time of diagnosis
Teratoma Histology: irregular, anastomosing vascular channels
• Pericardial teratoma is a benign germ cell lined by atypical endothelial cells (with abundant
cytoplasm).
neoplasm that typically affects infants and
children
• Presents with respiratory distress
and cyanosis secondary to pericardial Angiosarcoma
tamponade and compression of right- • Pericardial invasion
sided vascular structures • Chamber impingement
• Microscopically, cardiac teratomas • Complications
are similar to benign extrapericardial  Tamponade
teratomas  Dysrhythmias
• Contain derivatives of all 3 germ layers,  Myocardial rupture
with mature endodermal, mesodermal,
and ectodermal elements Angiosarcoma [Figure 12]
• Mass arising in RA wall
Malignant Cardiac Tumors – Sarcomas • Heterogeneous (hemorrhagic)
• Malignant pericardial effusion
Cardiac Sarcomas • Pulmonary mets (30%)
• Rare malignant mesenchymal neoplasms
• Majority of primary malignant cardiac
neoplasms
• Second most common primary cardiac
tumor
• Most common cell types
 Angiosarcoma (37% of cases)
 Unclassified or undifferentiated
sarcoma (24%)
 Malignant fibrous histiocytoma (MFH)
(11%–24%)
 Leiomyosarcoma (8%–9%)
 Osteosarcoma (3%–9%)

Primary Cardiac Masses Cardiac Imaging


1528
Osteosarcoma [Figures 13 & 14]
Figure 12 • Rare
Mass arising from
• Almost exclusively in LA
the myocardium • Calcium (+/-)
of the right • Tend to invade pulm vein
atrium – may be • Mets often at presentation
homogeneous or  Lung, nodes, thyroid, skin
heterogeneously
enhancing
depending on
tumor contents.
A malignant-
appearing
pericardial
effusion, which Figure 13
may demonstrate
pericardial A 50-year-
thickening, old woman
nodules, higher presented
Hounsfield units with cough
than simple fluid. and dyspnea,
These tumors history
tend to present of breast
late, and given their location on the right Ht, often carcinoma.
patients present with pulmonary metastases in about
1/3 of cases. These met lesions tend to be ill-defined
due to their hemorrhagic histologic character.

Angiosarcoma
• Originates in RA wall
• Heterogeneous signal (T1, T2)
• Nodular areas of increased intensity
 Thrombus, hemorrhage
• +/- pericardial thickening, nodules

Angiosarcoma – Magnetic Resonance


• Infiltrative mass + necrosis + hemorrhage
 T1-weighted image – large
heterogeneous mass coming off
the RA, high signal is mainly from
hemorrhage
 T1-weighted image postgadolinium –
large heterogeneous enhancing mass,
necrotic regions that do not enhance
with gadolinium.
Figure 14 A, B & C
Cardiac Sarcomas A 50-year-old woman presented with cough and
• Primary cardiac sarcomas are highly dyspnea, history of breast carcinoma. Dark blood
aggressive lesions images, pre- and post- gadolinium axial MR (top right)
 Uniformly fatal and axial MR post gad (bottom right) images.
 Mean survival of affected patients
approximately 3 months to 1 year
 Even after complete tumor excision, Malignant Cardiac Tumors –
local recurrence and metastatic Rhabdomyosarcoma
disease occur frequently and early 
usually within 1 year

Malignant Cardiac Tumors –


Osteosarcoma

Cardiac Imaging Primary Cardiac Masses


1529
Rhabdomyosarcoma [Figure 15] Figure 16
• Most common primary cardiac malignancy
of childhood Secondary
lymphoma
• No specific chamber predilection
(usually spread
• Arise from myocardium, more likely to of non-Hodgkin
affect valves than other sarcomas lymphoma)
• Two types: is much more
 Embryonal (more common; child or common than
primary – in fact,
adults) it is the second
 Pleomorphic (less common, adults) most common
• Local recurrence common metastatic
 Myocardium, pericardium malignancy to the
heart (after lung
carcinoma). About
1/4 of all patients
with disseminated
lymphoma,
almost always
non-Hodgkin
lymphoma,
eventually
have cardiac
involvement.
Primary cardiac lymphoma is defined as extranodal
lymphoma, exclusively located in the heart and
pericardium, and is almost always B-cell and most
Figure 15 A & B often occurs in human immunodeficiency virus
patients. Primary cardiac lymphoma almost always
A 22-year-old woman. Axial T1 MR 6 months after develops in right heart, usually right atrium. Usually
tumor debulking shows the common event of presents with large pericardial effusion.
RECURRENCE along interatrial septum and pericardial
space (left image). At lower level, there is also tumor
along anterior free wall of right ventricle.

Malignant Cardiac Tumors – Lymphoma

Primary Cardiac Lymphoma


• Typically of the non-Hodgkin type
• Secondary much more common than
primary
 1/4 patients with disseminated
lymphoma Figure 17 A & B
• Second most common metastasis to heart
An 80-year-old man presented with progressive
dyspnea – chest X-ray shows enlarged cardiac
Lymphoma [Figure 16]
silhouette – ? Pericardial effusion. Usually presented
• Primary = “extranodal non-Hodgkin with large pericardial effusion (pericardial cytology
lymphoma” (RARE) often diagnostic). Right ventricle mass (anterior wall).
 B-cell
 Human immunodeficiency virus (HIV)
 Right heart (RA)
• Histology: lymphoma cells surrounding,
infiltrating residual myocytes

Lymphoma: Imaging [Figures 17 to 19]


• Mural mass in RA>RV>LV>LA
• Chamber impingement
• SSuperior vena cara (SVC), inferior vena Figure 18 A, B & C
cara (IVC) occlusion
• Pericardial invasion TYPICAL A 36-year-old man presented with superior vena
 Massive effusion cava syndrome (this lesion propagated well into the
 Cytology diagnostic 67% superior vena cava). Notice collateral venous flow in
mediastinum.

Primary Cardiac Masses Cardiac Imaging


1530
Primary Cardiac Tumors – Special
Features
• Myxoma:
 LA, endocavitary
• Papillary fibroelastoma:
 Valvular
• Lipoma:
 Fat-saturated MR confirms
• Rhabdomyosarcoma
 Children, young adults
• Angiosarcoma
 RA
• Osteosarcoma
 LA
• Lymphoma:
 Right heart + pericardial effusion

Cardiac Masses – Clinicopathological


Features
Figure 19 A, B, C & D • Best prognosis
A 36-year-old man presented with superior vena
 “Congenital” benign tumors
cara syndrome (this lesion propagated well into the ™™ Lipoma
superior vena cara) – notice collateral venous flow in ™™ Rhabdomyoma
mediastinum. Bright blood axial MR with filling defect ™™ Hemangioma
in the right atrium and post gadolinium MR shows ™™ Ventricular fibroma
variable enhancement.
• Poorest prognosis
 Metastatic disease to the heart
Secondary malignancies  Primary cardiac sarcomas
• Secondary malignancies with cardiac
involvement ~20–40 times more frequent
than primary cardiac neoplasms
• Autopsy studies, patients with known
malignant neoplasms will have cardiac
metastatic involvement in 10%–12% of
cases

Metastatic Disease
eukemia
Lung carcinoma
• (#1) Melanoma
Extracardiac
sarcoma
• Lymphoma (#2) Breast CA
Renal CA
Thyroid CA

Simple Approach to Cardiac Tumors


• Most cardiac tumors are BENIGN
 1. Myxoma
 2. Lipoma
 3. Fibroma
• Common primary cardiac MALIGNANCIES
 1. Adult – Angiosarcoma
 2. Pediatrics – Rhabdomyosarcoma
• Most malignant cardiac tumors are
METASTATIC

Cardiac Imaging Primary Cardiac Masses


1531
Case Seminars: Cardiac

Jean Jeudy, MD
Case 1: Question… Which is the best Case 3: Question… Which is the best
diagnosis? [Figure 1] diagnosis? [Figure 3]
• A. Aortic dissection • A. Myocardial infarction
• B. Intramural hematoma • B. Ebstein anomaly
• C. Penetrating aortic ulcer • C. Tricuspid insufficiency
• D. Aortic mycotic aneurysm • D. Arrhythmogenic right ventricular
cardiomyopathy

Figure 3

Figure 1 A & B

Case 2: Question… Which of the


following is the most appropriate Case 4: Question... Myocardial delayed
diagnosis [Figure 2] enhancement with this percentage
• A. Myocardial infarct of transmural involvement would
• B. Myocarditis be an acceptable Cut Off to consider
• C. Myocardial stunning revascularization?
• D. Myopia • A. No more than 25% of myocardial
thickness
• B. No more than 50% of myocardial
thickness
• C. No more than 75% of myocardial
thickness
• D. Any infarct seen by delayed
enhancement precludes consideration for
revascularization

T2-weighted STIR Viability imaging DHE


Figure 2 A & B

Question… Which of the following


regarding positive coronary
remodeling is false?
• A. Results from alterations of the arterial
wall secondary to atherosclerosis
• B. MDCT is more sensitive in evaluating
coronary remodeling compared to
angiography
• C. Associated with unstable (vulnerable)
plaque
• D. Resulting luminal narrowing leads to
increased risk for occlusion
Cardiac Imaging Case Seminar: Cardiac
1533
Case 5: Question... Regarding
ventricular noncompaction, which
of the following statements is true?
[Figure 4]
• A. Rare disorder classically demonstrating
abnormal thickening of the right ventricle
• B. There are characteristic ECG changes
• C. Primarily an acquired condition as a Figure 6
result of previous viral illness
• D. Complications include heart failure,
arrhythmias, and thrombosis

Case 8: Question... How would You


Characterize the finding? [Figure 7]
• A. Coronary anatomy bypass; saphenous
to diagonal branch
• B. Anomalous coronary anatomy; most
common form
• C. Anomalous coronary anatomy; strong
Figure 4 A & B
correlation with sudden death
• D. Anomalous coronary anatomy; inferior
Case 6: Question... Which of the ischemia possible
following best characterizes the
previous finding? [Figure 5]
• A. Coronary spasm
• B. Postinflammatory coronary atresia
• C. Reconstruction artifact
• D. Myocardial bridging

Figure 7

Case 9: Question... Which of the


following Has The worst acute
Figure 5 A & B
prognosis?
• A. Bicuspid aortic valve
• B. Drug-induced tricuspid valvulopathy
Case 7: Question... Which would be the
• C. Infarcted papillary muscle with rupture
most appropriate diagnosis? [Figure 6]
• D. Multivalvular rheumatic fever
• A. Multivessel ischemia
• B. Myocarditis Case 10: Question... Which is the best
• C. Arrhythmogenic right ventricular diagnosis? [Figure 8]
cardiomyopathy • A. Myocardial infarction
• D. Hemochromatosis • B. Ebstein anomaly
• C. Tricuspid insufficiency
• D. Arrhythmogenic right ventricular
cardiomyopathy

Case Seminar: Cardiac Cardiac Imaging


1534
Figure 8 A & B

Case 11: Question... Which is the best


answer? [Figure 9]
• A. Myocardial infarction
• B. Myocarditis
• C. Hypertrophic cardiomyopathy
• D. Arrhythmogenic right ventricular
cardiomyopathy

Figure 9 A & B

Cardiac Imaging Case Seminar: Cardiac


1535

You might also like